Anda di halaman 1dari 137

CARA JITU MENGUASAI

OLIMPIADE MATEMATIKA
UNTUK SMP
z

0 x

Bimmo Dwi Baskoro, S.Si.

Jakarta, 26 Maret 2012


KATA PENGANTAR

Buku ini dirancang untuk melengkapi siswa-siswi SMP dengan penalaran konsep dasar
serta kemahiran dalam menyelesaikan soal-soal olimpiade matematika yang sifatnya tidak rutin.
Perlu disadari bahwa tidak semua materi soal yang muncul pada kompetisi sekelas Olimpiade
Matematika tercakup dalam kurikulum regular SMP. Oleh karena itu diperlukan upaya lebih
besar dalam mengenalkan soal Olimpiade Matematika dengan berbagai solusi yang sifatnya
dapat merangsang siswa untuk berfikir secara kreatif.

Buku ini diharapkan dapat dipelajari untuk digunakan sebagai alat bantu dalam
menghadapi kompetisi matematika khususnya olimpiade matematika. Rincian pembahasan
dalam buku ini terdiri atas soal olimpiade tingkat Kabupaten / Kota, Provinsi dan Nasional, serta
lampiran Canadian Mathematics Olympiad. Penulis sengaja memisahkannya supaya siswa dapat
dengan mudah mempelajari buku ini secara bertahap. Kemudian setelah siswa dibekali taktik dan
strategi pemecahan masalah, penulis sertakan pula latihan soal tanpa pembahasan di akhir bab
untuk mengevaluasi pemahaman siswa. Sasaran yang ingin dicapai setelah siswa mempelajari
buku ini dengan baik adalah,

 Memperoleh pengetahuan dasar dan pola pikir bermatematika;


 Memperoleh daya nalar dan kreatifitas yang tinggi setelah diberikan taktik dan strategi
dalam pemecahan soal olimpiade matematika;
 Dapat dengan mudah menerjemahkan suatu kasus ke dalam bahasa matematika;
 Siswa mendapatkan prestasi yang tinggi dalam kompetisi matematika khusususnya dalam
olimpiade matematika.

Penulis menyadari bahwa dengan segala keterbatasan dan kompleksitas dalam pengerjaan
buku ini, tentu saja masih terdapat kekurangan. Oleh karena itu masukan dari pembaca sangat
penulis hargai dan penulis tunggu di bimmo.dwi@gmail.com.com. Dengan segala kelebihan dan
kekurangannya, penulis berharap semoga buku ini bermanfaat bagi pembaca.

Jakarta, April 2015

Penulis

.
DAFTAR ISI

Kata Pengantar ………………………………............................................................................................. i

Daftar Isi ………………………………………………………………………………………………………………………………………. ii

BAGIAN I TINGKAT KABUPATEN / KOTA ……….……………………………………………………………………........... 1

Petunjuk ……………………………………………………………………………………………………………………………………….. 1

Soal Pembahasan I …………………………………………………………………………………………………..……………………. 2

Soal Pembahasan II ………………………………………………………………………..……………………………………………… 22

Latihan I ………………………………………………………………………………………………………………………………………… 37

Latihan II ……………………………………………………………………………………………………………………………………….. 43

BAGIAN II OLIMPIADE TINGKAT PROVINSI/NASIONAL ……………………………………………………………………. 48

Petunjuk tahap I …………………………………………………………………………………………………………………………….. 48

Soal Pembahasan Tahap I ………………………………………………………………………………………………………………. 49

Petunjuk tahap II …………………………………………………………………………………………………………………………… 73

Soal Pembahasan Tahap II ……………………………………………………………………………………………………………… 74

Latihan Tahap I ………………………………………………………………………………………………………………………………. 86

Latihan Tahap II …………………………………………………………………………………………………………………………….. 90

BAGIAN III TINGKAT NASIONAL.. ……………..……………. ..…………………………………………………………………… 93

Soal Pembahasan …………………………………………………………………………………………………………………………. 93

Latihan …………………………………………………………………………………………………………………………………………. 105

BAGIAN IV LAMPIRAN CANADIAN MATHEMATICAL OLIMPIADE ……………………………………………………. 106

Fryer Contest ……………………………………………………………………………………………………………………………….. 106

Pascal Contest ………………………………………………………………………………………………………………………………. ?

Daftar pustaka ………………………………………………………………………………………………………………………………. ?


BAGIAN I TINGKAT KABUPATEN / KOTA

OLIMPIADE SAINS TINGKAT KABUPATEN / KOTA


DIREKTORAT PENDIDIKAN LANJUTAN PERTAMA
DIREKTORAT JENDRAL PENDIDIKAN DASAR DAN MENENGAH
DEPARTEMEN PENDIDIKAN NASIONAL
BIDANG STUDI: MATEMATIKA

PETUNJUK

1. Ada 2 jenis soal yang perlu anda jawab di dalam lomba ini, yaitu soal
pilihan ganda dan soal isian singkat

2. Untuk soal pilihan ganda (bobot 1)


a. Pilihlah jawaban yang paling benar dari pilihan yang tersedia.
b. Berdasarkan pilihan tersebut, silanglah huruf yang bersesuaian padan
lembar jawaban.
c. Jika anda mengubah jawaban yang sudah terlanjur anda lakukan,
lingkari tanda silang yang salah dan silanglah jawaban yang seharusnya.

3. Untuk soal isian singkat (bobot 2)


a. Isilah pada lembar yang disediakan jawabannya saja (tidak perlu
prosesnya).
b. Kalau memerlukan satuan ukuran, berikan pula satuan ukurannya.

4. Waktu yang disediakan untuk menjawab semua soal ini adalah 2 jam (2 × 60
menit) tanpa istirahat.
SOAL PEMBAHASAN I

DEPARTEMEN PENDIDIKAN NASIONAL


OLIMPIADE MATEMATIKA SMP
TINGKAT KABUPATEN / KOTA

SOAL

PILIHAN GANDA

1. Berapa digit terakhir dari (2002)2002?


a. 4
b. 2
c. 8
d. 0
e. 1

2. Suatu pentagon mempunyai sudut-sudut yang sama. Pentagon seperti pada


gambar dikelilingi oleh lima persegi dan lima segitiga. Berapakah besar sudut x
pada pentagon seperti yang diperlihatkan pada gambar?
x

a. 75o
b. 108o
c. 90o
d. 720
e. 750

3. Jika a, b, dan c adalah tiga biilangan bulat positif berbeda yang memenuhi abc =
16, berapakah nilai terbesar yang mungkin dari ab – bc + ca?
a. 253
b. 63
c. 249
d. 263
e. 200
4. Seseorang pengendara mobil dalam suatu perjalanan, mempunyai catatan jarak
(km) dan waktu (jam) yang ditempuh sebagai berikut.

Waktu 07.30 08.00 08.30 09.00 09.30 10.00


Jarak 0 60 100 100 150 200

Berapakah kecepatan rata-rata mobil tersebut?

a. 40 km/jam
b. 60 km/jam
c. 80 km/jam
d. 35 km/jam
e. 30 km/jam

5. Jika diberikan suatu barisan bilangan 3, 5, 9, 15, 23, …, berapakah suku ke-16?
a. 212
b. 243
c. 214
d. 178
e. 170

6. Dua puluh empat anak dapat menyelesaikan suatu pekerjaan dalam 90 jam.
Setelah mereka bekerja selama 46 jam, mereka istirahat selama 12 jam. Jika
pekerjaan tersebut harus selesai pada waktunya, berapa banyak anak harus
ditambah?
a. 6
b. 9
c. 11
d. 5
e. 7

7. Jika X = {a, b, c} dan Y = {1,2} maka himpunan pasangan berurutan dari X × Y


adalah …
a. {(2, a), (2, b), (2, c), (a, 1), (b, 1), (c, 1)}

b. {(a, 2), (b, 2), (c, 2), (1, a), (1, b), (1, c)}

c. {(1, a), (1, b), (1, c), (2, a), (2, b), (2, c)}

d. {(a, 1), (a, 2), (b, 1), (b, 2), (c, 1), (c, 2)}

e. {(a, 1), (a, 2), (b, 2), (b, 1), (c, 2), (c, 3)}
8. Perhatikan gambar roda seperti pada gambar. Panjang jari-jari roda 22 cm dan
tebal roda 6 cm. Apabila roda tersebut menggelinding lurus 7 kali putaran dan π
22
= , berapakah panjang lintasan roda tersebut?
7

22 cm

a. 968 cm
b. 1.137 cm
c. 1.232 cm
d. 924 cm
e. 824 cm

9. Berapakah luas daerah yang diarsir pada gambar?


1 cm

1 cm

1 cm 1 cm

1 cm 1 cm 1 cm

15 3
a.   cm2
2 2
17 3
b.   cm2
2 4
9 3
c.   cm2
2 4
7 3
d.   cm2
2 4
7
e. cm2
2

10. Empat bilangan pertama dari barisan bilangan segitiga adalah


1, 3, 6, 10, …
Berapakah bilangan segitiga ke-10?
a. 55
b. 45
c. 66
d. 78
e. 70

ISIAN SINGKAT

1. Misalkan titik A terletak pada garis L yaitu y = 5x + 3. Koordinat titik B adalah


(3,-2). Jika T adalah titik tengah dari AB sehingga AB tegak lurus dengan garis L,
tentukan persamaan garis yang sejajar L dan melalui titik T!

2. Suatu angkutan kota mempunyai aturan pembayaran sebagai berikut. Pada saat
naik setiap penumpang harus membayar Rp. 600, setelah 4 km pertama harus
membayar Rp. 1.400, dan setiap menempuh 2 km berikutnya harus membayar Rp.
150 dan membayar Rp 100 setelah 1 km. Budi menaiki angkutan kota tersebut
sejauh 21 km, berapakah Budi minimal harus membayar jasa angkutan kota
tersebut?

3. Lingkaran dengan pusat A berjari-jari 3 dan lingkaran dengan pusat B berjari-jari


1 seperti pada gambar. Berapakah jarak dari O ke D?
y

A
B
x
O C

4. Perhatikan gambar di bawah ini! Segitiga ABC sama kaki dengan AB = AC dan
BC = 30 cm. Persegi EFGH mempunyai panjang sisi 12 cm. Berapakah luas
segitiga AEF?
A

E F

B G H C

5. Jika gambar di bawah ini menunjukan lipatan untuk membuat kubus, huruf
apakah yang berhadapan dengan huruf G?
K

I J S

G H

6. Dalam bujur sangkar ajaib seperti pada gambar, jumlah angka pada setiap baris,
kolom dan diagonal adalah sama. Berapakah jumlah tiga angka dari sembarang
barisnya?
2x 3 2

-3

0 x

7. Jika panjang sisi persegi ABCD 1 cm, berapakah luas bangun yang diarsir?
D C

A B
8. Jika x dan y dua bilangan positif dan rata-rata dari 4, 20, dan x adalah sama
dengan rata-rata dari y dan 16. Berapakah rasio dari x dan y?

9. Jika -2 ≤ x ≤ 5, -3 ≤ y ≤ 5, 4 ≤ z ≤ 8 dan w = xy – z, berpakah nilai terkecil dari w


yang memenuhi?

1 3
10. Suatu daerah dibatasi oleh persamaan y = 2x + 2, y = x + 1 dan y =  x + 7.
2 4
Berapakah nilai maksimum y pada daerah tersebut?

PEMBAHASAN

PILIHAN GANDA

1. Perhatikan bahwa digit terakhir dari 20022002 sama dengan digit terakhir dari 22002 .
Kemudian perhatikan bilangan 2n di mana n adalah bilangan asli.
21 = 2
22 = 4
23 = 8
24 = 16
25 = 32
26 = 64

Ternyata sifat dari digit terakhir pada bilangan 2n berulang dengan periode 4.
Artinya digit terakhir pada 25 sama dengan digit terakhir pada 21 , digit terakhir
pada 26 sama dengan digit terakhir pada 22 , begitulah seterusnya.
Dengan demikian kita dapat merumuskan bahwa
Digit terakhir dari 2( 4k + 1 ) adalah 2,
Digit terakhir dari 2( 4k + 2 ) adalah 4,
Digit terakhir dari 2( 4k + 3 ) adalah 8 dan
Digit terakhir dari 2( 4k + 4 ) adalah 6.
Dimana 4k adalah kelipatan 4 untuk k = 0, 1, 2, 3, …
Kemudian karena digit terakhir dari 2002n sama dengan digit teraklhir dari 2n
maka dapat disimpulkan bahwa
Digit terakhir dari 2002( 4k + 1 ) adalah 2
Digit terakhir dari 2002( 4k + 2 ) adalah 4
Digit terakhir dari 2002( 4k + 3 ) adalah 8 dan
Digit terakhir dari 2002( 4k + 4 ) adalah 6
Akan dicari digit terakhir dari 20022002.
20022002 = 2002( 4 × 500 + 2 )
= 2002( 4k + 2 ).
Jadi, digit terakhir dari 20022002 adalah 4
Jawaban (a)

2. Pandang segi 5 beraturan pada bangun di bawah ini!


D E

X
B
C
A
O

Kita bagi bangun segi 5 di atas menjadi, 5 buah segitiga yang sama, masing-
masing seperti segitiga AOB. Karena kelima segitiga tersebut sama, maka
 AOB =  BOC
3600

5
 720
Perhatikan bahwa segitiga AOB adalah segitiga sama kaki dimana AO = BO.
Akibatnya  ABO =  BAO
1800  720
=
2
0
108
=
2
= 540
Oleh karena itu  OBC = 540.
 ABO +  OBC +  ABD +  CBE +  X = 3600
 540 + 540 + 900 + 900 +  X = 3600
 2880 +  X = 3600
  X = 3600 - 2880
= 720
Jawaban (d)

3. Jika abc = 16 dan a, b dan c adalah bilangan bulat positif (bilangan asli) yang
berbeda, maka a, b dan c masing-masing haruslah merupakan faktor positif yang
berbeda dari 16.
Jadi, bilangan-bilangan yang dipebolehkan untuk a, b dan c adalah faktor positif
dari 16 yaitu 1, 2, 4, 8 dan 16.
Tabel berikut adalah daftar semua kemungkinan a, b dan c yang berbeda sehingga
abc = 16.
(a) (b) (c) ( ab ) ( bc ) ( ca ) ( ab - bc + ca )
1 2 8 1 256 8 -247
1 8 2 1 64 2 -61
2 1 8 2 1 64 65
2 8 1 256 8 1 249
8 1 2 8 1 256 263
8 2 1 64 2 1 63

Jadi, nilai terbesar yang mungkin dari ab - bc + ca adalah 263


Jawaban (d)

Jarak tempuh total


4. Kecepatan rata- rata =
Waktu total
200 km
=
2,5 jam
= 80 km/jam
Jawaban (c)

5. Sebelum membahas soal, akan dipelajari terlebih dahulu Barisan Aritmatika.


Barisan Aritmatika adalah suatu barisan yang mempunyai sifat
(Suku ke-2) – (Suku ke-1) = (Suku ke-3) – (Suku ke2) = … = (Suku ke-n) – (Suku
ke-(n-1)).
(Suku ke-n) – (Suku ke-(n-1)) biasa disebut dengan b (beda).
Jika banyaknya suku ada n buah, maka Barisan Aritmatika dapat disajikan sebagai
U1 , (U1 + b) , (U1 + 2b) , … , (U1 + (n - 1)b).
Jika setiap suku pada barisan aritmatika dijumlahkan, maka akan membentuk
deret aritmatika dan jumlah n suku pertamanya disebut Sn.
Misalkan Sn adalah jumlah n suku pertama dan Un adalah suku ke-n.
Secara umum, bentuk deret aritmatika adalah
Sn = U1 + U2 + … + U(n - 1) + Un
Karena selisih setiap 2 suku yang berurutan sama dan kita nyatakan selisihnya itu
sebagai b atau beda, maka
U2 = U1 + b
U3 = U2 + b = (U1 + b) + b = U1 + 2b
U4 = U3 + b = (U1 + 2b) + b = U1 + 3b
Dengan melihat keteraturan di atas, kita bisa merumuskan nilai Un.
Un = U1` + (n - 1)b
Perhatikan kembali deret
Sn = U1 + U2 + … + U( n – 1 ) + Un
= U1 + (U1 + b) + … + (U1 + (n-2)b) + (U1 + (n-1)b)
Tugas kita sekarang adalah menjumlahkan setiap 2 suku dengan aturan sebagai
berikut
Suku ke-1 dijumlahkan dengan suku ke-n, suku ke-2 dijumlahkan dengan suku
ke-(n -1), suku ke-3 dijumlahkan dengan suku ke-(n -2) dan seterusnya.
Sn = (U1 + Un) + (U2 + U( n – 1 )) + …
Perhatikan bahwa banyaknya suku sekarang menjadi, setengah dari banyaknya
suku sebelumnya. Selain itu, nilai dari setiap suku sekarang menjadi, sama yaitu
sama dengan U1 + Un.
Sehingga
n
Sn = (U1 + Un)
2
Sekarang kita bahas soal no. 4.
Perhatikan barisan berikut ini!
3 5 9 15 23 ...

2 4 6 8

2 2 2
Barisan di atas bukan barisan aritmatika karena bedanya tidak konstan (tetap).
Namun coba perhatikan bahwa jarak antar bedanya konstan yaitu 2.
U1 = 3
U2 = U1 + 2 = U1 + 1.2
U3 = U2 + 4 = U1 + 2 + 4 =U1 + 1.2 + 2.2
U4 = U3 + 6 = U1 + 1.2 + 2.2 + 3.2

Un = U1 + 1.2 + 2.2 + 3.2 + … + (n - 1).2


= 3 + [ 1.2 + 2.2 + 3.2 + … + (n - 1).2 ]
[ 1.2 + 2.2 + 3.2 + … + (n - 1).2 ] merupakan deret aritmatika dengan beda 2 dan
banyak sukunya (n - 1) buah.
[ 1.2 + 2.2 + 3.2 + … + (n - 1).2 ] = [2 + 4 + 6 + … + (n - 1). 2]
 n -1 2 + n -1 2
=
2
   
 n -1 2n
=  
2
= n (n - 1)
Jadi, Un = 3 + n(n - 1)
= n2 – n + 3
Sehingga U16 = 162 – 16 + 3
= 243
Solusi alternatif

Jika beda pertamanya tidak konstan sedangkan beda ke-2 nya konstan, dengan
menggunakan formula di atas, sudah bisa dipastikan bahwa rumus ke-n nya (Un)
merupakan polinom (suku banyak ) berderajat 2.
Jadi, kita bisa mencari solusi alternatif sebagai berikut
Un = an2 + bn + c
U1 = a + b + c = 3 … (1)
U2 = 4a + 2b + c = 5 … (2)
U3 = 9a + 3b + c = 9 … (3)

Dari (1) dan (2) diperoleh


4a + 2b + c = 5
a+ b+ c=3
------------------- --
3a + b =2 … (4)

Dari (1) dan (3) diperoleh


9a + 3b + c = 9
a+ b+ c=3
------------------- --
8a + 2b = 6 atau 4a + b = 3 … (5)

Dari (4) dan (5) diperoleh


4a + b = 3
3a + b = 2
------------ --
a =1
Dari persamaan (4) diperoleh
b = 2 – 3a
=2–3
=-1
Dari persamaan (1) diproleh
3=3–a–b
=3–1+1
=3
Jadi, Un = n2 – n + 3

Catatan :
Dengan cara yang sama kita bisa mencari formula Un pada barisan yang
mempunyai beda konstannya pada beda ke-3 (beda pertama dan keduanya tidak
konstan). Barisan yang seperti itu mempunyai formula
Un = an3 + bn2 + cn + d
Jadi, untuk mengetahui nilai a, b ,c dan d setidaknya perlu diketahui 4 persamaan
berbeda.
Jawaban (b)

6. Perhatikan tabel di bawah ini!

(No) (Jumlah Anak) (Waktu (Jam))


1 24 90
2 48 45
3 12 180

Apabila kita cermati, ternyata semakin banyak jumlah anak akan semakin sedikit
waktu yang diperlukan begitupun sebaliknya. Hal ini biasa disebut sebagai
perbandingan berbalik nilai. Sehingga kita bisa menuliskan hubungan tabel No. 1
dan 2 sebagai berikut
1 1 48 24
48 :  24 :  
45 90 1 1
45 90
48 24
 
1 1
45 90
48 24
 
90 45
Bentuk terakhir adalah suatu pernyataan yang benar.
Kembali ke permasalahan pada soal, ke 24 anak menginginkan pekerjaan selesai
tepat waktu yaitu 90 jam (termasuk waktu istirahat selama 12 jam). Karena
mereka telah bekerja selama 46 jam, maka sisa waktu menyelesaikan tepat waktu
adalah (90 - 46) jam = 44 jam, dengan catatan mereka tidak beristirahat. Karena
mereka beristirahat selama 12 jam, akibatnya sisa waktu menjadi, (44 - 12) jam =
32 jam, oleh karena itu harus ditambah jumlah anak.
Misalkan x adalah banyaknya anak yang ditambahkan, maka tabelnya sebagai
berikut.

(Jumlah anak) (Waktu (jam))


24 44
(24 + x) 32
Menurut aturan perbandingan berbalik nilai, diperoleh
24 24 + x 24 24 + x
=  =
1 1 32 44
44 32
3 24 + x
 =
4 44
 24 + x = 33
x =9
Jadi, jumlah anak yang harus ditambahkan adalah 9 orang.
Jawaban (b)

7. Himpunan pasangan terurut dari X × Y adalah himpunan semua (x, y) di mana x


 X dan y  Y.
Jadi, jika X = { a, b, c} dan Y = {1, 2} maka himpunan pasangan terurut dari X ×
Y adalah {(a, 1), (a, 2), (b, 1), (b, 2), (c, 1), (c, 2)}
Jawaban (d)

8. Karena mempunyai ketebalan 6 cm, maka jari-jari total roda adalah (22 + 6) cm =
28 cm.
Jika roda menggelinding 7 kali putaran artinya roda tersebut menempuh jarak
sejauh 7 kali keliling roda.
Misalkan K adalah keliling total roda, maka
7K = 7( 2  r )
22
= 7 (2)( )(28)
7
= 1.232
Jawaban (c)

9. Perhatikan gambar berikut ini!


D C

1 cm I

1 cm

1 cm II 1 cm

A B
1 cm 1 cm 1 cm

Luas daerah yang diarsir = (Luas persegi ABCD) – (3 × Luas bangun I ) – (Luas
Bangun II)
1 1
= [32 – 3(  12 ) - (1)(3) ] cm2
4 2
3 3
= [9 -  - ] cm2
4 2
15 3
= [   ] cm2
2 4
Jawaban (a)

10. Pandang barisan segitiga pada soal sebagai barisan bilangan yang setiap sukunya
merupakan jumlah titik pada bilangan segitiga tersebut. Seperti pada soal No. 5,
barisan 1, 3, 6, 10, … mempunyai beda yang tidak konstan, masing-masing yaitu
2, 3, 4, …, tetapi jarak antar bedanya tetap yaitu 1.
U1 = 1
U2 = U1 + 2 = 1 + 2
U3 = U2 + 3 = 1 + 2 + 3
U4 = U3 + 4 = 1 + 2 + 3 + 4

Un = 1 + 2 + 3 + 4 + … + n
Bentuk terakhir di atas merupakan deret aritmatika dengan beda b = 1
n
Un = 1+ n 
2
Jadi,
10
U10 = 1  10 
2
= 5(11)
= 55
Soal ini bisa juga diselesaikan dengan solusi alternatif seperti pada soal No. 4
pilihan ganda.
Jawaban (a)

ISIAN SINGKAT

1. Perhatikan gambar di bawah ini!


y

Garis L
3 y = 5x + 3

Garis M
-1 1 3 x
0
A
T

C -2 D B

Misalkan garis yang sejajar dengan garis L dan melalui titik T itu disebut sebagai
garis M.
Dari gambar di atas, kita mempunyai segitiga siku-siku ABC dengan sudut siku-
siku di A. Karena T tepat terletak di tegah garis AB dan garis M memotong garis
AB di T, maka garis M juga akan memotong garis BC di D dengan BD = CD.
Sekarang akan dicari panjang BC.
Karena nilai y di B dan di C sama (yaitu y = -2) dan garis y = 5x + 3 (garis L)
melalui titik C,
Maka -2 = 5x + 3  5x = -5
 x = -1
Jadi, BC = 3 – (-1)
=4
1
Akibatnya CD = BC
2
1
= (4)
2
=2
Jadi, garis M adalah garis L yang digeser sebesar 2 satuan ke kanan yaitu
y = 5(x - 2) + 3  y = 5x – 7

2. Berikut adalah rincian pembayaran Budi jika jarak yang ditempuh adalah 21 km.
 Pembayaran saat naik = Rp. 600
 Pembayaran 4 km pertama = Rp. 1.400
16
 Pembayaran 16 km berikutnya = Rp. 1.200 (yaitu × Rp. 150)
2
 Pembayaran 1 km berikutnya = Rp. 100
----------------------------------------------------------- +
 Pembayaran 21 km (total) = Rp. 3.300

3. Perhatikan gambar di bawah ini!

H
J
A
B
E

O F G C

AB = (Jari-jari lingkaran besar) + (Jari-jari lingkaran kecil)


=3+1
=4
AE = AF – EF
= AF – BG
=3–1
=2
BE = AB2 - AE 2
= 42  22
= 16  4
= 12
=2 3
Perhatikan bahwa segitiga ABE dan ABH kongruen (semua sudut dan sisi yang
bersesuaian sama).
Misalkan  ABE =  ABH
=
AE
Maka sin  =
AB
2
=
4
1
=
2
= sin 300
Jadi,  = 300
Akibatnya  EBH = 2 
= 600
Karena CI dan BH sejajar, begitu juga CF dan BA sejajar,
maka  GCJ =  EBH
= 600
Akibatnya  GCB = 300.
Perhatikan juga bahwa segitiga ACF dan ABE sebangun, sehingga berlaku
CF AF CF 3
=  =
BE AE 2 3 2
 CF = 3 3
CO = CF + FO
= 3 3 +3
DO DO
tan 600 =  3=
CO 3 3 +3
 DO = 3 3  9 .

4. Perhatikan gambar di bawah ini!


A

E F
I

B H 12 cm G C

30 cm

BH = CG
30  12
=
2
= 9 cm
Perhatikan bahwa segitiga AFI dan FCG sebangun
1
dimana FI = GH
2
= 6 cm
AI FI AI 6cm
Akibatnya =  =
FG CG 12cm 9cm
 AI = 8 cm
1
Luas segitiga AEF = (EF)(AI)
2
1
= (12 cm)(8 cm)
2
= 48 cm2

5. Jika J ditetapkan sebagai sisi alas, maka H, I, K dan S adalah sisi-sisi samping
sedangkan G adalah sisi atas. Jadi, J adalah sisi yang berhadapan dengan sisi G.

6. Perhatikan gambar di bawah ini!

2x 3 2

-3

0 x y

Jika baris ke-3 kolom ke-3 diisi dengan y, maka


0+x+y=2–3+y  x=-1
Substitusikan x = -1 pada baris pertama
2x + 3 +2 = - 2 + 3 + 2
=3
Jadi, jumlah angka pada setiap baris, kolom maupun diagonalnya sama dengan 3.
Jika kita lengkapi semua kotak pada bujur sangkar ajaib tersebut, akan diperoleh

-2 3 2

5 1 -3

0 -1 4

7. Perhatikan gambar berikut ini!


D 1 cm C

1 cm

A F B
1
Bangun ABD = BAC merupakan bangun lingkaran. Misalkan E adalah titik
4
perpotongan busur lingkaran AC dan AC .
Karena AB, AE dan BE semuanya jari-jari lingkaran yang sama, maka segitiga
ABE sama sisi.
Oleh karena itu  BAE =  ABE
= 600
Akibatnya  DAE =  CBE
= 900 – 600
= 300
Luas bangun yang diarsir = (Luas persegi ABCD) - (Luas 2 × juring ADE ) –
(Luas segitiga ABE)
 Luas persegi ABCD = 1 cm2
30
 Luas 2 × juring ADE = 2( π(1cm 2 ) )
360
1
=  cm2
6
1
 Luas segitiga ABE = (AB)(EF)
2
1
= (AB) AE 2 - AF2
2
1 1
= (1 cm)( 1  cm)
2 4
1
= 3 cm2
4
 3
Luas bangun yang diarsir = ( 1   ) cm2
6 4

4 + 20 + x y +16
8. =  8 + 40 + 2x = 3y + 48
3 2
 2x + 48 = 3y + 48
 2x = 3y
x 3
 =
y 2
 x : y = 3: 2

9. Diketahui
-2 ≤ x ≤ 5
-3 ≤ y ≤ 5
4≤z≤8
w = xy - z
Perhatikan bahwa w bernilai minimum jika xy bernilai minimum dan z bernilai
maksimum.
Karena x dan y keduanya memiliki nilai positif dan negatif, maka agar xy
mencapai nilai paling minimum syaratnya x dan y harus berbeda tanda. Pada saat
x = 5 dan y = -3 terJadi, xy paling minimum, yaitu xy = -15.
Kemudian karena z harus maksimum maka haruslah z = 8.
Jadi, nilai w minimum adalah
w = xy – z
= 5(-3) – 8
= -23

10. Pertama-tama akan digambar daerah yang dibatasi ketiga persamaan tersebut.
y

C
y = -¾ x + 7

y = 2x +2
B

A y=½x+1
x
O

Untuk mencari nilai maksimum dari daerah ABC, cukup dengan melihat titik dari
daerah ABC yang tertinggi.
3
Dari persamaan y = - x + 7 dan persamaan y = 2x + 2 diperoleh
4
3 11
- x + 7 = 2x + 2  x = 5
4 4
20
x=
11
 20 
Jadi, y = 2x + 2  y = 2   + 2
 11 
62
y=
11
Jadi, nilai maksimum dari daerah yang dibatasi oleh ketiga persamaan tersebut
62
adalah .
11
SOAL PEMBAHASAN II

DEPARTEMEN PENDIDIKAN NASIONAL


OLIMPIADE MATEMATIKA SMP
TINGKAT KABUPATEN / KOTA

SOAL

PILIHAN GANDA

1. 50502  49502  ...


a. 10
b. 100
c. 1.000
d. 10.000
e. 100.000

2. Diketahui persegi panjang ABCD berukuran 9 cm × 5 cm. Hanya DGHJ yang


bukan merupakan persegi pada persegi panjang ABCD tersebut. Berapa luas
daerah DGHJ?
D G F C
J
H I

A E B

a. 1,5 cm2
b. 2 cm2
c. 3 cm2
d. 3,5 cm2
e. 4 cm2

b
3. Jika a = maka b dinyatakan dalam a adalah ...
1- b
a. b = 1+ a 2
1+ a 2
b. b =
a2
a2
c. b =
1+ a 2
1- a 2
d. b =
a2
a2
e. b =
1- a 2

n(n +1)
4. Bilangan segitiga adalah bilangan yang berbentuk , dengan n adalah
2
bilangan asli. Berapakah banyaknya bilangan segitiga yang kurang dari 100?
a. 8
b. 9
c. 10
d. 13
e. 15

5. Joko mengalikan tiga bilangan prima berbeda sekaligus. Ada berapa faktor
berbeda dari bilangan yang dihasilkan?
a. 3
b. 4
c. 5
d. 6
e. 8

6. Persegi ABCD pada gambar di bawah ini memiliki luas 1 satuan luas. AE = BE
dan BE = BF. Pecahan yang menyatakan luas dari daerah DEF adalah ... satuan
luas.
D C

A E B

1
a.
3
2
b.
5
3
c.
5
3
d.
7
3
e.
8

s
7. Pecahan adalah pecahan sejati, jika s < t dan faktor persekutuan terbesar dari s
t
dan t adalah 1. Jika t memiliki nilai mulai dari dari 2 sampai dengan 9, dan s
bilangan bulat positif, maka banyaknya pecahan sejati berbeda yang dapat dibuat
adalah ...
a. 26
b. 27
c. 28
d. 30
e. 36

8. 3 % dari 81 sama dengan 9 % dari ...


a. 27
b. 54
c. 72
d. 90
e. 243

9. Jumlah 101 bilangan bulat berurutan adalah 101. Berapakah bilangan bulat
terbesar di dalam barisan bilangan tersebut?
a. 51
b. 56
c. 100
d. 101
e. 150

10. Dengan menggunakan uang koin Rp. 50, Rp. 100 dan Rp. 200, ada berapa
carakah kita menyatakan uang sebesar Rp. 2.000?
a. 20
b. 65
c. 95
d. 106
e. 121
ISIAN SINGKAT

1. Perhatikan gambar di bawah ini!


P Q

X
S R

Diketahui  SPT = 830 dan  PQT = 410. Garis PQ dan RS sejajar, demikian
juga garis PS dan QT sejajar. Berapakah besar  X?

2. Alex selalu berbohong pada hari-hari Kamis, Jumat dan Sabtu. Pada hari-hari lain
Alex selalu jujur. Di lain pihak, Frans selalu berbohong pada hari-hari Minggu,
Senin dan Selasa, dan selalu jujur pada hari-hari lain. Pada suatu hari, keduanya
berkata : "Kemarin saya berbohong". Hari mereka mengucapkan perkataan
tersebut adalah hari …

n +3
3. Semua n sehingga n dan keduanya merupakan bilangan bulat adalah …
n -1

1 2 3 11
4. Misalkan N = + 2 + 3 +... + 11 . Dalam bentuk desimal, nilai dari N
10 10 10 10
adalah ...

5. Diberikan tempat air berbentuk kerucut (lihat gambar di bawah). Untuk mengisi
1
air sampai pada ketinggian t diperlukan air sebanyak 38,5 liter. Berapa liter lagi
2
air yang diperlukan untuk memenuhi tempat tersebut?

t
t/2

6. 213 jika dibagi dengan 13 akan memberikan sisa sama dengan ...

7. Tujuh ekor kambing menghabiskan rumput seluas 7 kali ukuran lapangan


sepakbola dalam waktu 7 hari. Waktu yang diperlukan oleh 3 ekor kambing untuk
menghabiskan rumput seluas 3 kali ukuran lapangan sepakbola adalah ... hari
8. Rata-rata sembilan bilangan adalah 6. Satu di antara kesembilan bilangan
1
dibuang. Rata-rata delapan bilangan yang tinggal adalah 6 . Bilangan yang
2
dibuang adalah ...

9. Banyaknya angka (digit) pada bilangan 22004 × 52003 adalah ...

10. Perhatikan gambar berikut!


D C
3
P
5
A B

Jika panjang BP = 160 , maka panjang CP = …

PEMBAHASAN

PILIHAN GANDA

1. Ingat bahwa a2 – b2 = (a + b)(a - b)


Maka
50502  49502  (5050  4950)(5050  4950)
 (10.000)(100)
 (104 )(102 )
 106
1
 (106 ) 2  103  1000

Jawaban (c)

2. Perhatikan gambar di bawah ini!


D G F C
J
H I

4 cm 5 cm

4 cm 5 cm
A E B

Luas DGHJ = (Luas ABCD) – (Luas BCFE) – (Luas AEIJ) – (Luas FGHI)
= (9 cm × 5 cm) – (5 cm)2 – (4 cm)2 – (1 cm)2
= 45 cm2 – 25 cm2 – 16 cm2 – 1 cm2
= 3 cm2
Jawaban (c)

b b
3. a =  a2 =
1- b 1- b
 a 2 1- b  = b
 a 2 - a 2b = b
 a 2b + b = a 2
 b  a 2 +1 = a 2
a2
b=
a 2 +1
Jawaban (c)

4. Misalkan n adalah bilangan asli.


Akan dicari banyaknya bilangan segitiga yang nilainya kurang dari 100.

n(n +1)
< 100  n  n +1 < 200
2

Nilai n terbesar yang memenuhi ketaksamaan di atas adalah n = 13.

Jawaban (d)

5. Bilangan prima adalah bilangan asli yang mempunyai tepat 2 faktor yaitu 1 dan
dirinya sendiri.
Misalkan ketiga bilangan prima yang dikalikan Joko masing-masing adalah X, Y
dan Z.
Karena ketiganya merupakan bilangan prima, maka X mempunyai faktor 1 dan X,
Y mempunyai faktor 1 dan Y serta Z mempunyai faktor 1 dan Z.
Akibatnya bilangan baru yang dihasilkan mempunyai faktor 1, X, Y, Z, XY, XZ,
YZ dan XYZ.
Jadi, terdapat tepat 8 faktor baru yang dihasilkan dari perkalian 3 bilangan prima
X, Y, dan Z.
Jawaban (e)

6. Perhatikan gambar di bawah ini!


D C

A E B

Luas ABCD = 1 Satuan Luas

AB × BC = 1 Satuan Luas
Karena AB = BC, maka
(AB)2 = 1 Satuan Luas
AB = BC = 1 Satuan Panjang
1
AE = BE = BF = CF = Satuan Panjang
2
Luas DEF = (Luas ABCD) – (Luas ADE) – (Luas DCF) – (Luas BEF)
1 1 1
= [1 - ( × AE × AD) – ( × CF × CD) – ( × BE × BF)] Satuan Luas
2 2 2
1 1 1 1 1 1 1
= [1 – ( × × 1) – ( × × 1 ) – ( × × )] Satuan Luas
2 2 2 2 2 2 2
1 1 1
= [1 - - - ] Satuan luas
4 4 8
3
= Satuan Luas.
8
Jawaban (e)

7. Kita kelompokkan untuk s = 1 sampai s = 9


 Untuk s = 1
s 1 1 1 1 1 1 1 1
Nilai yang mungkin dari adalah , , , , , , dan
t 2 3 4 5 6 7 8 9
Terdapat 8 pecahan sejati.
 Untuk s = 2
s 2 2 2 2
Nilai yang mungkin dari adalah , , dan
t 3 5 7 9
Terdapat 4 pecahan sejati.
 Untuk s = 3
s 3 3 3 3
Nilai yang mungkin dari adalah , , dan
t 4 5 7 8
Terdapat 4 pecahan sejati.
 Untuk s = 4
s 4 4 4
Nilai yang mungkin dari adalah , dan
t 5 7 9
Terdapat 3 pecahan sejati
 Untuk s = 5
s 5 5 5 5
Nilai yang mungkin dari adalah , , dan
t 6 7 8 9
Terdapat 4 pecahan sejati.
 Untuk s = 6
s 6
Nilai yang mungkin dari adalah
t 7
Terdapat 1 pecahan sejati.
 Untuk s = 7
s 7 7
Nilai yang mungkin dari adalah dan
t 8 9
Terdapat 2 pecahan sejati.
 Untuk s = 8
s 8
Nilai yang mungkin dari adalah
t 9
Terdapat 1 pecahan sejati.

Jadi, seluruhnya terdapat 27 pecahan sejati.

Jawaban (b)

8. 3 % × 81 = 9 % × Y (kedua ruas dibagi dengan 3 %)


81 = 3Y
Y = 27
Jawaban (a)

9. Misalkan suku pertama dari bilangan yang berurutan itu adalah X.


X + (X + 1) + (X + 2) + … + (X + 100) = 101
Ruas kiri dari persamaan terakhir merupakan deret aritmatika dengan
b=1
n = 101
U1 = X
Un = U101 = X + 100
Jadi, X + (X + 1) + (X + 2) + … + (X + 100) = Sn
n 101
Sn = (U1 + Un)  S101 = (X + X + 100)
2 2
101
 101 = (2X + 100)
2
 101 = 101 (X + 50) (Kedua ruas dibagi 101)
 1 = X + 50
 X = - 49
Jadi, barisan itu adalah -49, -48, …, 50, 51
Oleh karena itu, bilangan terbesar pada barisan itu adalah 51.
Jawaban (a)

10. Banyak cara untuk menyatakan uang Rp. 2.000 dengan menggunakan koin Rp.
50, Rp. 100 dan Rp. 200.
 Dengan uang Rp. 50 saja.
Terdapat 1 cara.
 Dengan uang Rp. 100 saja
Terdapat 1 cara.
 Dengan uang Rp. 200 saja.
Terdapat 1 cara.
 Dengan uang Rp. 50 dan Rp. 100
Yaitu dengan memakai sebuah uang Rp. 100 , 2 buah uang Rp. 100
sampai memakai 19 buah uang Rp. 100.
Jadi, terdapat 19 cara berbeda.
 Dengan uang Rp. 50 dan Rp. 200
Yaitu dengan memakai sebuah uang Rp. 200, 2 buah uang Rp. 200
sampai 9 buah uang Rp. 200.
Jadi, terdapat 9 cara berbeda.
 Dengan uang Rp. 100 dan Rp. 200
Yaitu dengan memakai sebuah uang Rp. 200, 2 buah uang Rp. 200
sampai 9 buah uang Rp. 200.
Jadi, terdapat 9 cara berbeda.
 Dengan uang Rp 50, Rp. 100 dan Rp. 200
 Mengandung sebuah uang Rp. 200
Sehingga uang bersisa Rp 1.800
Uang Rp. 1.800 dapat dinyatakan dengan cara memakai sebuah
uang Rp. 100, 2 buah uang Rp. 100 sampai 17 buah uang Rp.
100
 Mengandung 2 buah uang Rp. 200
Sehingga uang bersisa Rp. 1.600
Uang Rp. 1.600 dapat dinyatakan dengan cara memakai sebuah
uang Rp. 100, 2 buah uang Rp. 100 sampai 15 uang Rp. 100
Dengan cara yang sama proses tersebut dilakukan sampai mengandung
9 uang Rp. 200,
Jadi, dengan uang Rp. 50, Rp. 100 dan Rp. 200 diperoleh (17 + 15 +
13 + … + 1) cara.
1 + 3 + … + 15 + 17 adalah deret aritmatika dengan
b=2
U1 = 1
Un = 17
Un = U1 + (n - 1)b = 17  1 + (n - 1)2 = 17
 1 + 2n – 2 = 17
 2n = 18
 n=9
n
Sn = (U1 + Un)
2
9
= (1 + 17)
2
= 81
Jadi, terdapat 81 cara yang berbeda.

Total cara menyatakan uang Rp. 2.000 oleh Rp. 50, Rp. 100 dan Rp. 200 adalah

1 + 1 + 1 + 19 + 9 + 9 + 81 = 121 cara berbeda.

Catatan :

Lihat pembahasan barisan aritmatika pada pembahasan soal pilihan ganda Versi

I tingkat Kabupaten / Kota.

Jawaban (e)

ISIAN SINGKAT

1. Perhatikan gambar berikut ini!


P Q

X
S
U R

Diketahui  SPT = 830 dan  PQT = 410.


Karena PQ dan RS sejajar begitu juga PS dan QR sejajar maka
 PSR =  PQT
= 410
Pada segitiga PSU,
 SPU +  PSU +  PUS = 1800  830 + 410 +  PUS = 1800
  PUS = 560
 PUS +  X = 1800  560 +  X = 1800
  X = 1240

2. Dengan cara inspeksi (memeriksa), hari yang tepat keduanya berkata: “Kemarin
saya berbohong” adalah hari Minggu.

n +3
3. Akan dicari setiap bilagan bulat n sehingga merupakan bilangan bulat.
n -1
n + 3 (n -1) + 4
=
n -1 (n -1)
4
=1+
n 1
4
Akibatnya harus bilangan bulat.
n -1
4
Supaya merupakan bilangan bulat, maka (n - 1) haruslah merupakan faktor
n -1
dari 4 yaitu -1, -2, -4, 1, 2 dan 4.
 Jika n- 1 = -1
Maka n = 0
 Jika n – 1 = -2
Maka n = -1
 Jika n – 1 = -4
Maka n = -3
 Jika n – 1 = 1
Maka n = 2
 Jika n – 1 = 2
Maka n = 3
 Jiaka n – 1 = 4
Maka n = 5
n +3
Jadi, semua bialangan bulat n yang mengakibatkan bilangan bulat adalah
n -1
-3, -1, 0, 2, 3 dan 5.

1 2 3 10 11
4. N = + 2 + 3 +... + 10 + 11
10 10 10 10 10
10 11
= 0,123456789 + 10  11
10 10
1 1,1
= 0,123456789 + 9  10
10 10
Jadi,
0,123456789
0,000000001
0,00000000011
---------------------- +
N = 0,12345679011

5. Soal ini dapat diselesaikan dengan 2 cara.

Cara I
1 1
Misalkan V1 adalah volume kerucut yang tingginya t dan jari-jari alasnya r,
2 2
sedangkan V2 adalah volume kerucut yang tingginya t dan jari-jar alasnya t.
2
1 1  1  1 2
V1 = π  r   t   38,5 liter = πr t
3 2  2  24
 πr 2 t = 924 liter
1 2
V2 = πr t
3
1
= × 924 liter
3
= 308 liter
Jadi, banyaknya air yang harus ditambahkan adalah
V2 – V1 = (308 – 38,5) liter
= 269,5 liter.
Cara II

Dengan cara yang praktis, kita bisa langsung membandingkan volum kerucut
besar dan kecil, karena keduanya sebangun.
2
1 1  1  1 2
π r t
V1 3  2   2 
πr t
38,5 liter 8
=  =
V2 1 2
πr t V 2 πr 2 t
3
38,5 liter 1
 =
V2 8
 V2 = 8  38,5 liter 
 V2 = 308 liter
Jadi, banyaknya air yang harus ditambahkan adalah
V2 – V1 = (308 – 38,5) liter
= 269,5 liter

6. Sebelum membahas soal, akan dipelajari terlebih dahulu materi tentang Kongruen
Modulo.
Bilangan bulat P kongruen dengan Q modulo n atau ditulis P  Q (mod n) apabila
P dan Q menghasilkan sisa yang sama jika keduanya dibagi oleh bilangan bulat n.
Contoh 5 dan 3 adalah kongruen modulo 2 atau ditulis 5  3 (mod 2) , karena 5
dan 3 keduanya bersisa 1 jika dibagi oleh 2.
Sifat bilangan kongruen modulo.
Misalkan a = (bn + c)m dengan n, m bilangan asli dan c bilangan bulat (perhatikan
bahwa c boleh merupakan bilangan negatif) maka berlaku
a  cm (mod n)
Contoh
5 = (1 × 2 + 3)1 = (2 × 2 + 1)1 maka 5  31 (mod 2)  11 (mod 2)
atau 5  3 (mod 2)  1 (mod 2)
Metode di atas berlaku juga untuk m > 1.
Sekarang akan dibahas soal No. 6
213 = 2( 4 × 3 + 1 )
= 2 × 2( 4 × 3 )
Perhatikan, mengapa 13 diuraikan menjadai (4 × 3 + 1)? Karena pembaginya 13,
Jadi, diusahakan harus mengandung suku yang mendekati 13 yaitu 24 = 16.
213 = 2 × (24)3
= 2 × (16)3
= 2 × (1 × 13 + 3)3
Karena (1 × 13 + 3)3  33 (mod 13) = 27 (mod 13) maka
213  2 × 27 (mod 13) = 2(2 × 13 + 1) (mod 13)  2 × 1 (mod 13)
Jadi, 213  2 (mod 13).
Artinya, 213 dan 2 jika dibagi 13 akan menghasilkan sisa yang sama. Karena 2
dibagi 13 sisanya 2, maka jika 213 dibagi 13 akan bersisa 2 juga.
Catatan : Perhatikan waktu penggunaan pemakaian lambing ‘ = ’ dan ‘  ‘ ,
karena keduanya memiliki makna yang berbeda.

7. Misalkan kesembilan bilangan itu adalah x1 , x 2 ,..., x 9


x1 + x 2 +... + x 9
Maka = 6  x1 + x 2 +... + x 9 = 54
9
Kemudian misalkan bilangan yang dibuang adalah x9
x1 + x 2 +... + x 8
Maka = 6,5  x1 + x 2 +... + x 8 = 52
8
Akibatnya
x1 + x 2 +...+ x8 + x 9 = 54
x1 + x 2 +...+ x8  52
------------------------------------ --
x9 = 2
Jadi, bilangan yang dibuang adalah 2.

8. 7 ekor kambing menghabiskan rumput seluas 7 kali ukuran lapangan sepakbola


dalam 7 hari.
Artinya
1 ekor kambing menghabiskan rumput seluas 1 kali ukuran lapangan sepakbola
dalam 7 hari.
Jadi,
3 ekor kambing menghabiskan rumput seluas 3 kali ukuran lapangan sepakbola
dalam 7 hari.

9. 22004× 52003 = 2 × 22003× 52003


= 2(2 × 5)2003
= 2(10)2003
Bentuk terakhir adalah suatu bilangan yang berdigit 2004 buah terdiri atas digit 2
di awal (sebanyak 1 buah) dan digit 0 untuk seterusnya (sebanyak 2003 buah).
Jadi, banyaknya digit pada bilangan 22004. 52003 adalah sebanyak 2004 buah.
Catatan:
Perhatikan bahwa kata ‘ banyaknya ’ dan ‘ jumlah ’ adalah berbeda. Sebagai
ilustrasi, banyaknya digit pada bilangan 2007 adalah 4 buah sedangkan jumlah
digit pada bilangan 2007 adalah (2 + 0 + 0 +7) = 9.
10. Perhatikan gambar di bawah ini!
D G C

3 P
H F

5 160

A E B
2 2 2 2
HP + AH = HP + EP = 25
HP2 + DH2 = HP2 + GP2 = 9
------------------ --
EP2 – GP2 = 16 … (1)
Kemudian
EP2 + BE2 = EP2 + CG2 = 160 … (2)
Dari persamaan (2) dan persamaan (1) diperoleh
EP2 + CG2 = 160
EP2 – GP2 = 16
--------------------- --
CG2 + GP2 = 144
Karena CG2 + GP2 = CP2 maka
CP2 = 144
CP = 12
Jadi, panjang CP = 12.
LATIHAN I

DEPARTEMEN PENDIDIKAN NASIONAL


OLIMPIADE MATEMATIKA SMP
TINGKAT KABUPATEN / KOTA

SOAL

PILIHAN GANDA

1. Titik E terletak di dalam persegi ABCD, dan segitiga ABE merupakan segitiga
sama sisi. Tentukan  BEC!
a. 30°
b. 60°
c. 70°
d. 75°
e. 80°

2. Dalam suatu segitiga ABC diketahui  A = 550,  C = 750, D terletak pada sisi
AB dan E pada sisi BC. Jika DB = BE, maka  BED = ...
a. 50°
b. 55°
c. 60°
d. 65°
e. 75°

1
3. Suatu keluarga mempunyai 4 orang anak, anak termuda berumur dari anak
2
tertua, anak kedua 3 tahun lebih tua dari anak yang termuda dan anak ketiga 5
tahun lebih muda dari anak yang tertua. Bila rata-rata umur mereka 16 tahun,
maka umur anak tertua adalah … tahun.
a. 16
b. 18
c. 20
d. 22
e. 24
4. Jika pembilang suatu pecahan ditambah 2 dan penyebutnya ditambah 3, maka
3
hasilnya . Jika pembilang dikurangi 1 dan penyebutnya ditambah 4, maka
4
1
hasilnya . Pecahan itu adalah …
3
2
a.
3
3
b.
5
3
c.
4
4
d.
5
5
e.
6

5. Sebuah saluran air seharusnya dibuat dengan menggunakan pipa berdiameter 10


cm. Akan tetapi yang tersedia hanyalah pipi-pipa kecil yang berdiameter 3 cm.
Supaya kapasitas saluran tidak lebih kecil daripada yang diinginkan, berapa
banyaknya pipa berdiameter 3 cm yang perlu dipakai sebagai pengganti 1 pipa
berdiameter 10 cm?
a. 3
b. 5
c. 7
d. 11
e. 12

6. Diketahui a + b = 1 dan a2 + b2 = 2. Nilai a4 + b4 = …


1
a. 3
2
b. - 3
1
c. 
2
1
d.
2
1
e. 3
2
1 1 4
7. Misalkan m dan n adalah bilangan positif yang memenuhi + = .
m n 7
Nilai m2 + n2 adalah …
a. 121
b. 200
c. 212
d. 232
e. 256

8. Banyaknya diagonal yang dapat dibuat pada sebuah segi banyak dengan 100 sisi
adalah …
a. 4.650
b. 4.750
c. 4.850
d. 4.950
e. 5.150

2
9. Jika perbandingan 2X – Y terhadap X + Y adalah , maka perbandingan X
3
terhadap Y adalah …
a. 1 : 5
b. 4 : 5
c. 1 : 1
d. 5 : 1
e. 5 : 4

10. Diketahui salah satu akar persamaan x2 – 5x + p = 0 adalah 2. Nilai p adalah …


a. -6
b. -3
c. -2
d. 2
e. 6

ISIAN SINGKAT

1. Panjang rusuk sebuah kubus 9 cm. Luas bola yang menyinggung sisi-sisi dalam
kubus adalah …

2. Jumlah 10 bilangan adalah 36 lebih besar dari rata-rata kesepuluh bilangan-


bilangan tersebut. Jumlah kesepuluh bilangan tersebut adalah …
3. Perhatikan gambar di bawah ini!
D

A B C

Jika panjang BC = 10 cm, besar  CBD = 450,  CAD = 300. Maka panjang AB
= … cm

4. Himpunan 20 buah bilangan mempunyai rata-rata 20. Sembilan di antara bilangan


tersebut rata-ratanya 9. Rata-rata dari 11 bilangan yang tersisa adalah …

5. Selisih 2 bilangan positif adalah 5, sedangkan jumlah kuadratnya 2.100 kurangnya


dari kuadrat jumlah kedua bilangan itu. Jumlah kedua bilangan tersebut adalah …

6. Selisih panjang rusuk dari 2 kubus adalah 2 cm. Selisih volumenya 218 cm3.
Panjang rusuk kubus yang besar adalah …

7. Diketahui keliling persegi ABCD = 112 cm.


22
(  )
7
D C

A B

Luas daerah yang diarsir adalah …

8. Anton mengendarai motor dari Kota X ke Kota Y pada pukul 09:30 dengan
kecepatan rata-rata 60 km/jam, sedangkan Tony mengendarai mobil dari Kota Y
ke Kota X pada pukul 10:15 dengan kecepatan 40 km/jam. Jika jarak Kota X dan
Y 345 km, pada pukul berapa kedua pengendara berpapasan?

9. Sebuah bak air di bawahnya terdapat 2 kran yang berukuran sama, sebut kran 1
dan kran 2. Jika kran 1 dan 2 dibuka, maka air akan habis dalam waktu 2 jam.
Berapa lama air akan habis jika kran 2 saja yang dibuka?
10. Diketahui segitiga ABC siku-siku di B dan AB = BC = 4 cm
C

A
B

Luas daerah yang diarsir adalah …


KUNCI JAWABAN

PILIHAN GANDA

1. d
2. d
3. d
4. d
5. e
6. e
7. b
8. c
9. e
10. e

ISIAN SINGKAT

1. 81  cm2
2. 40
3. 10  
3 1
4. 29
5. 65
6. 7 cm
7. 448 cm2
8. 13:15
9. 6 jam
10. 4   2  cm 2
LATIHAN II

DEPARTEMEN PENDIDIKAN NASIONAL


OLIMPIADE MATEMATIKA SMP
TINGKAT KABUPATEN / KOTA

SOAL

PILIHAN GANDA

1. Besar pelurus sudut A adalah 4 kali penyikunya, maka besar sudut A adalah …
a. 72°
b. 84°
c. 104°
d. 108°
e. 60°

2. Panjang sisi miring segitiga siku-siku adalah 15 cm dan kelilingnya 36 cm. Luas
segitiga tersebut adalah … cm2
a. 24
b. 48
c. 54
d. 90
e. 108

3. Seorang anak merahasiakan 3 buah bilangan, kemudian ia menjumlahkan setiap 2


bilangan itu dan hasilnya sama dengan 15, 17, dan 20. Jumlah ketiga bilangan itu
adalah …
a. 25
b. 26
c. 27
d. 30
e. 35

4. X dan Y bersama-sama menyelesaikansuatu pekerjaan memerlukan waktu 4 jam


48 menit. Jika X menyelesaikan pekerjaan sendiri, memerlukan waktu 8 jam.
Waktu yang diperlukan Y untuk menyelesaikan pekerjaan dengan sendiri adalah
… jam.
a. 8
b. 10
c. 12
d. 14
e. 16

5. Matematikawan August Morgan menghabiskan seluruh usianya pada tahun 1.800-


an. Pada tahun terakhir dalam masa hidupnya dia mengatakan bahwa: “Dulu aku
berusia P tahun pada tahun P2”. Pada tahun berapa ia dilahirkan?
a. 1806
b. 1822
c. 1849
d. 1851
e. 1853

6. Pada suatu segitiga ABC, sudut C adalah 3 kali lebih besar dari sudut A dan sudut
B adalah 2 kali leih besar dari sudut A. Berapa perbandingan antara panjang sisi
AB dan BC?
a. 1 : 1
b. 1 : 2
c. 2 : 3
d. 2 : 1
e. 3 : 2

7. Misalkan:
A = Segitiga sama kaki
B = Segitiga sama sisi
C = Persegi panjang
D = Persegi
E = Lingkaran
Jika keliling kelima bangun di atas sama, maka bangun yang terluas adalah …
a. A
b. B
c. C
d. D
e. E

8. Akar-akar persamaan kuadrat x2 – 13x + 36 = 0 adalah P2 dan Q2. Nilai dari P +


Q=…
a. 2
b. 3
c. 5
d. 6
e. 13

9. Dari 25 orang yang melamar suatu pekerjaan diketahui bahwa 7 orang berumur
lebih dari 30 tahun dan 15 orang bergelar sarjana. Di antara pelamar yang bergelar
sarjana 5 orang berumur lebih dari 30 tahun. Banyaknya pelamar yang bukan
sarjana dan umurnya kurang dari 30 tahun adalah … orang.
a. 5
b. 6
c. 7
d. 8
e. 9

10. Pedagang ayam mempunyai 6 ekor ayam jantan dan 4 ekor ayam betina. Ia akan
menjual 5 ekor dari ayamnya. Berapa peluang yang terjual 3 diantaranya ayam
betina?
5
a.
21
10
b.
21
1
c.
70
1
d.
40
3
e.
40

ISIAN SINGKAT

1. Dalam trapesium ABCD, sisi AB dan CD sejajar. Diagonal BD dan sisi AB sama
panjang. Jika  BCD = 110° dan  CBD = 30°, maka  BAD = …
D C

A B

 1  1  1  1  1  1 
2. 1  1  1  1  1  1    ...
 3  4  5  6  7  8 
3. Luas sisi sebuah balok berturut turut 9 cm2, 6 cm2, dan 3 cm2. Panjang diagonal
ruang balok tersebut adalah … cm.

4. X dan Y merupakan bilangan asli yang memenuhi sistem persamaan:


X2 + 2XY = 40
1
Y2 + XY = 15
2
Niali X2 – Y2 = …

1
5. Selisih uang A dan B adalah Rp. 60.000. Jika A memberikan uangnya kepada
5
B, maka uang mereka menjadi sama. Berapa jumlah uang mereka mula-mula?

6. Lima ekor kambing memakan rumput sebanyak 5 keranjang dalam 5 hari. Berapa
hari yang diperlukan oleh 3 ekor kambing untuk menghabiskan rumput sebanyak
3 keranjang?

7. Sebuah bola tenis dijatuhkan dari ketinggian 7,5 m dan selalu mematul kembali
4
dengan ketinggian kali tinggi semula. Pantulan terJadi, terus menerus sampai
5
bola berhenti. Jumlah seluruh lintasan yang ditempuh oleh bola adalah …

22
8. Diketahui keliling lngkaran 176 cm2 dimana  = dan sudut BAC = 450.
7
C

Luas daerah yang diarsir adalah …

9. Sebuah limas T. ABCD alasnya berbentuk persegi dengan keliling alas 64 cm.
Jika panjang setiap rusuk yang lainnya adalah 17 cm, maka luas permukaan limas
adalah …

10. Suatu kerucut diameter alasnya 20 cm dan volumnya 2.512 cm3. Berapa luas
22
permukaan kerucut bila  = ?
7
KUNCI JAWABAN

PILIHAN GANDA

1. e
2. c
3. b
4. c
5. a
6. d
7. e
8. c
9. d
10. a

ISIAN SINGKAT

1. 700
1
2.
4
7
3. 2
2
4. 7
5. Rp. 240.000
6. 5 hari
7. 67,5 m
8. 224 cm2
9. 544 cm2
10. 1.069 cm2
BAGIAN II TINGKAT PROVINSI
OLIMPIADE SAINS TINGKAT PROVINSI
DIREKTORAT PENDIDIKAN LANJUTAN PERTAMA
DIREKTORAT JENDRAL PENDIDIKAN DASAR DAN MENENGAH
DEPARTEMEN PENDIDIKAN NASIONAL
BIDANG STUDI : MATEMATIKA

PETUNJUK
TES ISIAN SINGKAT (TAHAP I)

1. Tes terdiri atas 25 soal. Waktu yang disediakan 150 menit.

2. Skor setiap butir soal yang dijawab benar adalah 1 dan bobot setiap soal
nilainya sama.

3. Tuliskan nama, asal sekolah dan nomor peserta anda di sebelah kanan atas
pada setiap lembar jawaban.

4. Tuliskan jawaban pada lembar jawaban yang telah disediakan.

5. Segala macam bentuk buram yang dilakukan oleh Anda harap dikumpulkan
pada pengawas untuk diJadi,kan tinjauan penilaian.

6. Jawaban hendaknya anda tuliskan dengan menggunakan tinta bukan pensil.

7. Selama tes, Anda tidak diperkenankan menggunakan buku, catatan dan alat
bantu hitung (kalkulator). Anda jugaaa tidak diperkenankan bekerjasama.

8. Mulailah bekerja hanya setelah pengawas memberi tanda dan berhentilah


bekerja segera setelah pengawas memberi tanda.

9. Ukuran pada gambar tidak mewakili pada ukuran sebenarnya.

10. Selamat bekerja.


SOAL PEMBAHASAN TAHAP I

TINGKAT PROVINSI

TAHAP I

SOAL ISIAN SINGKAT

1. Nilai dari 3
82  ...
a 2 + 4ab + b 2
2. Didefinisikan bahwa (a * b) = . Maka nilai dari 4*(3)  ...
a+b

3. Sisa pembagian dari 7100 oleh 16 adalah …

4. Yanti menghabiskan Rp. 2.000 untuk membeli 3 bungkus kacang dan 4 bungkus
keripik, sedangkan Triana membeli 6 bungkus kacang dan 2 bungkus kripik
dengan menghabiskan Rp. 2.350 pada warung Anda. Maka harga sebungkus
keripik adalah …

5. Diketahui a dan b adalah bilangan asli dimana faktor persekutuan terbesar dari a
a
dan b adalah 3, dan = 0, 4 . Hasil dari ab = ...
b

6.

[1] [2] [3]

Diberikan pola barisan dari kiri ke kanan. Banyak bulatan pada gambar ke-n
adalah ...

7. Suatu keluarga mempunyai 5 orang anak. Anak termuda berumur setengah dari
anak tertua sedang 3 anak lainnya berturut-turut berumur lebih dari 2 tahun dari
termuda, lebih 4 tahun dari termuda dan kurang 3 tahun dari tertua. Bila rata-rata
hitung umur mereka adalah 16, maka umur anak tertua adalah ...

8. Sejumlah siswa di sekolah X harus menyumbang uang sebanyak Rp. 96.000.


Setiap siswa harus memberi iuran yang sama. Kemudian ternyata bahwa 4 siswa
tidak membayar. Untuk menutupi kekurangannya siswa yang lainnya harus
menambah iurannya masing-masing Rp. 2.000. Jumlah siswa yang membayar
adalah ...

9. Dari 100 siswa, 39 diantaranya gemar olahraga. Diantara penggemar olah raga
tersebut 11 siswa juga gemar bermain musik. Jika ternyata 32 siswa tidak gemar
olah raga maupun musik, maka banyaknya penggemar musik di antara 100 siswa
tersebut adalah ...

10. Bila penduduk Jawa Tengah adalah 25 % dari penduduk pulau Jawa dan 15 %
dari penduduk Indonesia. Penduduk Indonesia yang tinggal di luar pulau jawa ...
%.

11. Gambar dibawah adalah penampang sebuah saluran air yang berbentuk lingkaran
dengan diameter 10 cm dan lebar permukaan airnya adalah 5 cm. Tinggi
permukaan air dan luas penampang air berturut-turut adalah ...

Lebar
tinggi

12. Di samping adalah gambar tiga


lingkaran dengan ukuran jari-jari
yang sama dan ketiganya saling
bersinggungan. Maka luas daerah
yang diarsir adalah ...
13. Ketika menghitung volum suatu tabung, Sunardi melakukan kesalahan. Ia memasukan
diameter alas ke dalam rumus volum tabung, padahal seharusnya jari-jari alasnya yang
dimasukan. Berapa rasio hasil perhitungan Sunardi terhadap hasil yang seharusnya?

14. Diketahui AB sejajar dengan EC. Jika  BAD = 75° dan  CDE = 50°, maka  BCE =

B

D
E A

15. Luas A, B dan C berturut-turut adalah 90 m2, 120 m2 dan 36 m2. Luas daerah D adalah …

B D

A C

16. Diketahui segitiga ABC siku-siku di B dengan AC = 40 cm dan BC = 24 cm. Titik D


terletak pada AB sedemikian sehingga CD = 25 cm. Jika AD = x, maka nilai x adalah ...
C

A D B

17. Pola ABCCDEABCCDEABCCDE ... berulang sampai tak terhingga. Huruf yang akan
menempati urutan ke 2753 adalah ...

7
18. Sebuah bola berjari-jari m menggelinding dari tembok A ke tembok B. Ternyata bola
22
itu menggelinding sebanyak 10 putaran. Berapa meter jarak antara tembok A dengan
tembok B?
A B

19. Dari segitiga ABC diketahui bahwa titik A adalah perpotongan garis 2x + y – 6 = 0 dan
garis 2x + 2y – 3 = 0. Koordinat B dan C berturut-turut adalah (0,1) dan (1,2). Tentukan
persamaan garis tinggi dari sudut A pada segitiga tersebut!

20. Dari 6 buah kartu bernomor 1 sampai 6 diambil dua kartu secara acak. Berapakah
peluang terambilnya dua kartu yang jumlah nomornya adalah 6?

1 1
21. Misalkan x dan y adalah bilangan real tidak nol. Diketahui + = 10 dan x + y = 40.
x y
Berapakah nilai xy?

22. Misalkan ABCD adalah sebuah persegi. Titik E terletak dalam persegi sedemikian
sehingga segitiga ABE sama sisi. Maka besar sudut dalam AEC pada segi-4 ADCE
adalah ...

a - (b - c)
23. Bentuk sederhana dari adalah ...
(a - b) 2 - c2

24. Suatu bujur sangkar sisinya 6 cm berputar pada titik O yang merupakan titik pusat
bujursangkar lain yang bersisi 4 cm. Luas bidang yang diarsir adalah ...
4 cm

6 cm

25. Sebuah segitiga sama sisi, sebuah lingkaran dan sebuah persegi memiliki keliling yang
sama. Dari ketiga bangun tersebut yang memiliki luas terbesar adalah ...
PEMBAHASAN

3
1. -82 = 3 -(23 )2
1
= (- (23 ) 2 ) 3
1
= (- (2) 6 ) 3
= - (2)2
=-4

a 2 + 4ab + b 2 42 + 4(4)(-3) + (-3)2


2. (a * b) =  (4*(-3)) =
a+b 4 + (-3)
= - 23

3. Akan dicari bilangan perpangkatan 7 yang nilainya mendekati kelipatan 16.


72 = 49 mendekati 3 × 16 = 48
7100 = 7(2 × 50)

= 4950

= (48 + 1)50

 150 (mod 16)

= 1 (mod 16)

Jadi, sisa pembagian dari 7100 oleh 16 adalah 1.

4. Misalkan Sebungkus kacang = x


Sebungkus keripik = y

3x + 4y = 2.000 … (1)

6x + 2y = 2.350 atau

3x + y = 1.175 … (2)

Dari persamaan (1) dan (2) diperoleh

3x + 4y = 2.000

3x + y = 1.175
-------------------- --

3y = 825

y = 275

Jadi, harga sebungkus keripik adalah Rp. 275.

a
5. Akan dicari suatu pecahan yang paling sederhana sehingga FPB dari a dan b adalah 1.
b
a
= 0, 4
b
2
=
5
Karena FPB dari a dan b harus 3, maka

a 2 3
= ×
b 5 3
6
=
15

Jadi, a = 6 dan b = 15

Akibatnya ab = 6(15)

= 90

6. Soal ini bisa diselesaikan dengan 2 cara.

Cara I

Jumlah titik pada gambar di soal mengikuti pola barisan


6, 10, 15, 21, ...

Perhatikan bahwa beda setiap suku tidak konstan yaitu

4, 5, 6, ...

Tetapi jarak antar bedanya konstan yaitu 1.

U1 = 6

U2 = U1 + 4
U3 = U2 + 5 = U1 + 4 + 5

U4 = U3 + 6 = U1 + 4 + 5 + 6

Un = U1 + 4 + 5 + 6 + … + (n + 2)

4 + 5 + 6 + … + (n + 2) adalah deret aritmatika dengan suku pertama 4, suku terakhir (n


+2), beda b = 1 dan banyaknya suku (n -1) buah.

 n -1
Jadi, 4 + 5 + 6 + … + (n + 2) =
2
 4 +  n + 2

 n -1
=  n + 6
2

n 2 5n
= + -3
2 2

n 2 5n
Un = U1 + + -3
2 2

n 2 5n
=6+ + -3
2 2

n 2 5n
= + +3
2 2

n 2 5n
Jadi, banyaknya bulatan hitam pada gambar ke-n adalah + +3.
2 2

Cara II

Karena setiap beda pertamanya tidak konstan sedangkan beda ke-2 nya konstan maka
formula untuk Un nya berupa polinom berserajat 2.

Un = an2 + bn + c

U1 = a + b + c = 6 … (1)

U2 = 4a + 2b + c = 10 … (2)

U3 = 9a + 3b + c = 15 … (3)

Dari persamaan (1) dan (2) diperoleh


4a + 2b + c = 10

a+ b+c= 6

--------------------- --

3a + b = 4 … (4)

Dari persamaan (1) dan (3) diperoleh

9a + 3b + c = 15

a+ b+c= 6

--------------------- --

8a + 2b = 9 … (5)

Dari persamaan (4) dan (5) diperoleh

8a + 2b = 9

6a + 2b = 8

--------------- --

2a =1

1
a =
2

Dari persamaan (4) diperoleh

b = 4 – 3a

3
=4-
2

5
=
2

Dari persamaan (1) diperoleh

c=6–a–b

1 5
=6- -
2 2
=3

Jadi, Un = an2 + bn + c

n 2 5n
= + +3
2 2

n 2 5n
Jadi, banyaknya bulatan hitam pada gambar ke-n adalah + +3.
2 2

7. Misalkan urutan umur dari yang tertua sampai yang termuda adalah

a, b, c, d dan e

1
e= a
2
1
d = e+2 = a+2
2
1
c = e+4 = a+4
2
b = a -3

Karena rata-rata hitung umur mereka 16, maka

a +b+c+d+e
= 16
5
1 1 1
a + (a - 3) + ( a + 4) + ( a + 2) + a
 2 2 2 = 16
5
7
 a + 3 = 80
2
7
 a = 77
2
 a = 22

Jadi, umur anak yang tertua adalah 22 tahun.

8. Misalkan n adalah banyaknya murid dan x adalah besarnya iuran yang dibebankan
kepada setiap murid.
Karena banyaknya murid yang tidak membayar adalah 4 orang, maka banyaknya murid
yang membayar adalah (n - 4) orang.
Jadi, model persamaan matematika yang bisa dibuat dari kasus ini adalah

96.000 = (n - 4)(x) + (n - 4)(2.000) ... (1)

Kemudian jumlah uang yang tidak dibayar oleh 4 murid itu harus ditanggung oleh (n - 4)
murid masing-nasing Rp. 2.000, maka:

4x = (n - 4)(2.000)  x = (n - 4)(500) …(2)

Dari persamaan (2) dan persamaan (1) diperoleh

96.000 = (n - 4)(n - 4)(500) + (n - 4)(2.000)

 96.000 = 500(n - 4)2 + 2.000(n - 4)

 192 = (n - 4)2+ 4(n - 4)

 (n - 4)2+ 4(n - 4) – 192 = 0

Misalkan (n - 4) = p, maka persamaannya menjadi,

 p2+ 4p – 192 = 0

 (p - 12)(p + 16)

 p = 12 atau p = -16

 n – 4 = 12 atau n – 4 = - 16

 n = 16 atau n = - 12

Karena banyaknya murid selalu positif, nilai yang memenuhi adalah n = 16.

Jadi, banyaknya murid yang membayar adalah (n - 4) = 12 orang.

9. Misalkan
O = Banyaknya siswa yang gemar olahraga.
M = Banyaknya siswa yang gemar musik.
X = Banyaknya siswa yang gemar musik saja.

O M

32 28 11 X
Karena total siswa 100 orang, maka:
32 + 28 + 11 + x = 100  71 + x = 100
 x = 29
Jadi, banyaknya siswa yang gemar musik saja adalah 29 orang
Akibatnya banyaknya siswa yang gemar musik adalah
M = 11 + x
= 11 + 29
= 40 orang.

10. Misalkan
X = Banyaknya penduduk Indonesia
Y = Banyaknya penduduk Jawa
Z = Banyaknya penduduk Jawa Tengah

X Y Z

Y 15%
Z = 25 % Y = 15 % X  =
X 25%
Y
 = 0,6
X
Y
 = 60 %
X
 Y = 60 % X
Jadi, banyaknya penduduk yang berada di luar Jawa adalah X - Y = X – 60 % X
= 40 % X
yaitu 40 % dari penduduk Indonesia.

11. Perhatikan gambar di bawah ini!


O

5 cm 5 cm

C
A B

Karena DO jari-jari, maka DO = 5 cm


CD = DO – CO
= DO - AO2 + AC2
  5  
2

= 5  5    cm
2

  2  

 125 
= 5   cm
 4 

 5 
= 5  3  cm
 2 
 1 
= 5 1  3  cm
 2 
Luas penampang air = Luas tembereng AB
= (Luas juring OAB ) - (Luas segitiga ABO)
Karena ABO adalah segitiga sama sisi, maka  AOB = 600.
60 2 1
= [ π  OA  -  AB  (CO) ] cm2
360 2
1 2 1
= [ π  OA  -  AB DO - CD  ] cm2
6 2
1 1 5 
= [ π  25  -  5  3  ] cm2
6 2 2 
25 25
= [ π- 3 ] cm2
6 4
25 π 3
= [ - ] cm2
2 3 2
 5 
Jadi, tinggi penampang air dan luas penampang air berturut –turut adalah  5  3  cm
 2 
25 π 3
dan [ - ] cm2.
2 3 2

12. Perhatikan gambar berikut ini!

C r r B

r r

r r

Karena ABC segitiga sama sisi, maka besar ketiga sudutnya adalah 600.
Kemudian pandang segitiga ABC!
C

2r 2r

A r r B
D

Luas daerah yang diarsir = (Luas Segitiga ABC) – (3 × Luas juring A )


1  60  2
=  AB  CD  - 3   πr
2  360 
1 1
= 2r 4r 2 - r 2 - πr 2
2 2
1
= r 2 3 - πr 2
2
 π
= r2  3 - 
 2
 π
Jadi, luas daerah yang diarsir adalah r 2  3 -  satuan luas.
 2
13. Hasil perhiitungan volum yang salah adalah π  2r  t = 4πr 2 t , sedangkan hasil
2

perhitungan volum yang sebenarnya adalah πr 2 t .


Jadi, rasio perhitungan volum yang salah terhadap perhitungan yang sebenarnya adalah
4πr 2 t
= 4 :1 .
πr 2 t

14. Perhatikan gambar pada soal!


Diketahui  BAD = 75°,  CDE = 50°
Karena segitiga ABD dan segitiga ECD sebangun maka  CED =  BAD = 750
akibatnya dari segitiga ECD diperoleh
 CED +  CDE +  DCE = 180°  75° + 50° +  DCE = 180°
  DCE = 55°
Kemudian karena  DCE dan  BCE saling berpelurus, maka:
 DCE +  BCE = 180°  55° +  BCE = 180°
  BCE + 55° = 180°
  BCE = 125°

15. Perhatikan gambar berikut ini!


R S

Q B D

P A C

90
Luas daerah A = P.R = 90  R = … (1)
P
120
Luas daerah B = Q.R = 120  R = … (2)
Q
Dari persamaan (1) dan (2) diperoleh

90 120 4
=  Q= P
P Q 3

36
Luas daerah C = P.S = 36  S =
P
 4  36 
Luas daerah D = Q.S =  P   = 48
 3  P 
Jadi, luas daerah D adalah = 48 cm2.

16. Perhatikan segitiga ABC di bawah ini!


C

40
24
25

A D y B
x

Misalkan BD = y, maka dari segitiga siku-siku BCD diperoleh


y= 252  242
= 625  576
= 49
=7
Kemudian dari segitiga siku-siku ABC diperoleh
x+y= 402  252  x + 7 = 1600  576
 x + 7 = 1024
 x + 7 = 32
 x = 25
Jadi, nilai x adala;h 25 cm.

17. Perhatikan bahwa pola ABCCDEABCCDEABCCDE … berulang setiap kelipatan 6


(mempunyai periode 6). Jadi, huruf yang menempati urutan ke- 2753 adalah huruf yang
menempati sisa pembagian 2753 oleh 6 pada deretan huruf ABCCDE. Untuk mencari sisa
pembagian dari bilangan yang besar, kita gunakan konsep kongruen modulo.

2753 = 2( 3. 2 + 1 ) 53
= 2(23)2 53
= 2 (8)2 5
= 2(6 + 2)2 (6 - 1)3
 2.22 (-1)3 (mod 6)
= - 8 (mod 6)
= - (6 + 2) (mod 6)
 - 2 (mod 6)
= - (6 - 4) (mod 6)
 - (- 4) (mod 6)
= 4 (mod 6)
Sisa pembagian 2753 oleh 6 adalah 4.
Urutan ke-4 dari deretan huruf ABCCDE adalah huruf C.
Jadi, karena 2753 kongruen dengan 4 (mod 6) maka huruf yang menempati urutan ke-
2753 adalah huruf C.

Catatan :
Untuk bilangan yang masih relatif kecil, kita bisa mencari sisa pembagian tersebut secara
langsung
2753 16.000

6 6
4
 2.666 
6
Jadi, sisa pembagiannya adalah 4.

18. Perhatikan gambar di bawah ini!

A C D B

AB = Jarak dari tembok A ke tembok B


AC = BD
= Jari-jari bola
7
= cm
22
CD = Jarak yang ditempuh bola
=10 Keliling
22 7
= 10(2) cm
7 22
= 20 cm

AB = 2AC + CD
7
= [ 2( ) + 20 ] cm
22
7
= 20 cm
11
7
Jadi, jarak tembok A ke tembok B adalah 20 cm.
11
19. Soal ini dapat diselesaikan dengan 2 cara.

Cara I

Perhatikan bahwa untuk menggambar segitiga ABC pada koordinat kartesius diperlukan
titik A yaitu perpotongan garis 2x + y - 6 = 0 dan garis 2x + 2y – 3 = 0. Dari kedua
persamaan garis tersebut diperoleh:

2x + 2y – 3 = 0
2x + y – 6 = 0
------------------ --
y+3=0  y=-3
2x + y – 6 = 0  2x = - y + 6
 2x = 3 + 6
 2x = 9
9
 x=
2
9
Sehingga titik A adalah ( , - 3)
2
Selanjutnya kita gambar segitiga ABC dengan titik B = (0, 1) dan titik C = (1, 2) pada
koordinat kartesius.
y

C
2
D

1 B

9/2
x
0 1

-3
A

Akan dicari persamaan garis AD yaitu persamaan garis yang tegak lurus dengan garis
BC.
Perhatikan bahwa persamaan garis BC adalah persamaan garis y = x yang digeser satu
satuan ke atas. Jadi, persamaan garis BC adalah y = x + 1.
Kemudian , kedua persamaan dikatakan tegak lurus jika perkalian gradiennya adalah – 1.
Misalkan gradien dari garis BC adalah m1 dan gradien dari garis AD adalah m2.
Dari persamaan garis BC diperoleh
y = x + 1 maka m1 = 1
m1.m2 = 1  m2(- 1) = 1
 m2 = - 1
9
Persamaan AD melalui titik (x1, y1) = ( , - 3) dan gradiennya m2 = - 1.
2
Suatu persamaan garis lurus yang melalui titik (x1, y1) dan gradien m2 dirumuskan
sebagai
9
y – y1 = m2(x – x1)  y – (- 3) = - 1(x - )
2
9
 y+3=-x+
2
 2y + 6 = - 2x + 9
 2x + 2y – 3 = 0
Jadi, persamaan garis AD adalah 2x + 2y – 3 = 0.

Cara II

Cara kedua adalah dengan jalan mencari koordinat titik D sehingga bisa dicari
persamaan garis AD, karena koordinat titik A sudah diketahui.
Pandang segitiga ABC pada gambar (cara I)

C B
D

2
9
   1  3
2
AB =
2
145
=
4
2
9 
  1   2  3
2
AC =
2 
149
=
4

 2  1  12
2
BC =
= 2
BC = BD + CD  2 = BD + CD
 CD = 2 - BD
Dari segitiga ACD diperoleh
149
AD2 = AC2 – CD2  AD2 = - ( 2 - BD)2 … (1)
4
Dari segitiga ABD diperoleh
145
AD2 = AB2 – BD2  AD2 = – BD2 … (2)
4
Dari persamaan (1) dan (2) diperoleh
149 145 149 145
- ( 2 - BD)2 = – BD2  - (BD2 - 2 2 BD + 2) = – BD2
4 4 4 4
 2 2 BD = 1
2
 BD =
4
(0, 2) (1, 2)
C

(0, 1) E
B

Karena garis BC membentuk sudut 450 terhadap garis BE, maka BE = DE


2 2
BE2 + DE2 = BD2  2BE2 = ( )
4
1
 2BE2 =
8
1
 BE2 =
16
1
 BE =
4
DE = BE
1
=
4
1 1 1 5
Jadi, titik D berada pada koordinat ( , 1 + ) = ( , )
4 4 4 4
9
Persamman garis AD yang melalui titik A = (x1, y1) = ( , - 3) dan titik D = (x2, y2) =
2
1 5
( , ) adalah
4 4
9
x-
y - y1 x - x1 y+3 2
=  =
y 2 - y1 x 2 - x1 5 1 9
+3 -
4 4 2
9
x-
y+3 2
 =
17 17
-
4 4
9
 y+3 = -x
2
 2y + 6 = 9 - 2x
 2x + 2y - 3 = 0
Jadi, persamaan AD adalah 2x + 2y – 3 = 0.

20. Misalkan xy melambangkan terambilnya kartu bernomor x dan kartu bernomor y. Jadi,
xy dan yx dipandang sebagai kasus yang sama.
Untuk mencari peluang, harus dicari semua ruang sampelnya di mana semua
kemungkinan 2 kartu terambil, yaitu
12, 13, 14, 15, 16
23, 24, 25, 26
34, 35, 36
45, 46
56
Banyaknya ruang sampel adalah 15.
Sedangkan ruang sampel yang jumlah nomornya 6 adalah
15 dan 24
Banyaknya ruang sampel yang jumlah nomornya 6 adalah 2
2
Jadi, peluang terambilnya 2 kartu yang jumlah nomornya 6 adalah .
15
1 1 x+y
21. + = 10  = 10
x y xy
40
 = 10
xy
 xy = 4

22. Perhatikan gambar berikut ini!


D C

A B

Karena segitiga ABE sama sisi, maka  ABE =  AEB =  BAE = 60°
 BAE +  DAE = 90°  60° +  DAE = 90°
  DAE = 30°
Karena AE = AD, maka segitiga AED sama kaki,
Akibatnya  AED =  ADE
1800  DAE
=
2

1800  300
=
2

1500
=
2

= 750

 CDE +  ADE = 90°   CDE + 75° = 90°


  CDE = 15°
Karena CE = DE, maka segitiga CDE sama kaki,
Akibatnya  DCE =  CDE
= 15°
 DCE +  CDE +  CED = 180°  15° + 15° +  CED = 180°
 30° +  CED = 180°
  CED = 150°
Jadi, sudut dalam AEC =  AED +  CED
= 75° + 150°
= 225°

a - (b - c)
23. Penyebut dari pecahan adalah bentuk x 2 - y2 = (x + y)(x - y)
(a - b) 2 - c2
a - (b - c) (a - b + c)
Jadi, 2 2
=
(a - b) - c (a - b + c)(a - b - c)
1
=
a -b-c

24. Perhatikan gambar di bawah ini!


D 4 cm C

H
O 6 cm
G
2 cm

A E F B

Segitiga GHO dan segitiga EFO kongruen karena GO = EO = 2 cm dan  EOF =


 GOH.
Luas FBHO = (Luas bangun FBGO) + (Luas segitiga GHO)
= (Luas bangun FBGO) + (Luas segitiga EFO)
= Luas persegi EBGO
= (EB)(BG)
= (2 cm)2
= 4 cm2

25. Misalkan K adalah keliling masing-masing dari segitiga sama sisi, persegi dan lingkaran.
 Untuk segitiga sama sisi
C

S S

A ½S D ½S B

1
K = 3S  S = K
3
1
Luas segitiga ABC = (AB)(CD)
2
2
1 S
2
= (S) S - 
2 2
SS 
=  3
22 
S2
= 3
4
2
K
 
=  
3
3
4
3 2
= K Satuan luas
36
 Untuk persegi
D S C

A B

1
K = 4S  S = K
4
Luas persegi ABCD = S2
1
= ( K)2
4
1 2
= K Satuan luas
16
 Untuk lingkaran

K
K = 2 r  r =

Luas lingkarean di atas = π r2
K2

4π 2
1
= K 2 Satuan luas

3 1 1
Karena   , maka bangun yang paling luas dengan keliling yang sama adalah
36 16 4
bangun lingkaran.
OLIMPIADE SAINS TINGKAT PROVINSI
DIREKTORAT PENDIDIKAN LANJUTAN PERTAMA
DIREKTORAT JENDRAL PENDIDIKAN DASAR DAN MENENGAH
DEPARTEMEN PENDIDIKAN NASIONAL
BIDANG STUDI : MATEMATIKA

PETUNJUK
TES URAIAN (TAHAP II)

1. Tes terdiri atas 15 soal. Waktu yang disediakan 120 menit.

2. Skor setiap butir soal yang dijawab benar adalah 2 dan bobot setiap soal nilainya
sama.

3. Tuliskan nama, asal sekolah dan nomor peserta anda di sebelah kanan atas pada
setiap lembar jawaban.

4. Tuliskan jawaban pada lembar jawaban yang telah disediakan.

5. Apabila kotak jawab yang disediakan tidak memenuhi, makaAnda dapat


meneruskannya di bagian belakang pada lembar soal dan jawaban.

6. Segala macam bentuk buram yang dilakukan oleh Anda harap dikumpulkan pada
pengawas untuk diJadi,kan tinjauan penilaian.

7. Jawaban hendaknya anda tuliskan dengan menggunakan tinta bukan pensil.

8. Selama tes, Anda tidak diperkenankan menggunakan buku, catatan dan alat bantu
hitung (kalkulator). Anda juga tidak diperkenankan bekerjasama.

9. Mulailah bekerja hanya setelah pengawas memberi tanda dan berhentilah bekerja
segera setelah pengawas memberi tanda.

10. Ukuran pada gambar tidak mewakili pada ukuran sebenarnya.

11. Selamat bekerja.


SOAL PEMBAHASAN TAHAP II

TINGKAT PROVINSI

TAHAP II

SOAL URAIAN

1. Hari ini adalah hari kamis. Jatuh pada hari apakah 2007 hari yang akan datang?

2. Sebotol sirup bisa digunakan untuk membuat 60 gelas minuman jika dilarutkan dalam air
dengan perbandingan 1 bagian sirup untuk 4 bagian air. Berapa gelas minuman yang
diperoleh dari sebotol sirup jika perbandingan larutan adalah 1 bagian sirup untuk 5
bagian air?

3 1
3. Dua pecahan jumlahnya dan selisihnya . Tentukan kedua pecahan tersebut!
4 12

4. Pilih sebuah bilangan 2 digit. Dengan membalikkan digitnya, dapat diperoleh sebuah
bilangan lain. Tunjukkan bahwa selisih kedua bilangan tersebut habis dibagi 9!

5. Gambar di bawah ini adalah sebuah trapesium. Buktikan bahwa c2 = a2 + b2!


R

S
c
b
c
a

P Q
b T a

6. Perhatikan gambar di bawah ini. Segitiga PQR adalah sebuah segitiga siku-siku di titik Q.
Jika panjang PQ = 16 cm, PR = 20 cm dan QS tegaklurus PR. Tentukan panjang garis
QS!
P

Q R

7. ABCD adalah sebuah persegi panjang dengan panjang AB = 3 cm dan BC = 2 cm. Jika
QD = 2 cm dan DP = 1 cm. Buktikan bahwa besar  BQP = 90°!
D Q C

A B

8. ABC adalah suatu segitiga dengan ukuran  BAC = 60°. Jika AF merupakan garis bagi
sudut BAC, tunjukkan bahwa segitiga yang diarsir merupakan sebuah segitiga sama sisi!
C

E
F

A B
D

9. Bangun ABC adalah segitiga siku-siku sama kaki. Jika PQRS merupakan sebuah persegi
yang luasnya 1 cm2, tentukan luas daerah segitiga ABC!
B

P Q

A C
S R

10. Lingkaran berpusat di titik asal dan berjari-jari 3 memotong sumbu x positif, sumbu y
positif dan sumbu y negatif berturut-turut di titik A, B dan C. Apabila dibuat garis
singgung di B sedemikian hingga garis yang melalui CA memotong garis tersebut di P.
Tentukanlah koordinat titik P!
11. Kota P dan Q berjarak 160 km. Misalkan A dan B melakukan perjalanan dari arah yang
berbeda. A dari Kota P menuju Kota Q dan B pada saat yang bersamaan melakukan
perjalanan dari Kota Q menuju Kota P. Jika A menggunakan sepeda dengan kecepatan
rata-rata 20 km/jam sedangkan B menggunakan motor dengan kecepatan rata-rata 60
km/jam. Setelah berapa lama mereka bertemu dan pada kilometer ke berapa dari Kota P
mereka bertemu?

12. Dari kelima kandidat Capres dan Cawapres Partai P, akan dipilih pasangan Presiden dan
Wakil Presiden. Berapa banyak pasangan yang mungkin terjadi?

13. Sebuah bak berbentuk kubus penuh air mempunyai luas bidang diagonal 9 2 cm2.
Kemudian dimasukkan sebuah kayu yang juga berbentuk kubus dengan volume 8 cm3
sehingga air tumpah. Hitung volume air yang tumpah, jika seluruh kayu tenggelam!

14. Diketahui segitiga ABC sama sisi dengan X pada BC sehingga CX = 2BX, perlihatkan
bahwa AX2 = 7BX2.
A

B X C

15. Pada perkalian di bawah ini, setiap huruf mewakili angka yang berbeda dan tunggal.
Berapa nilai A + B?
A 7
5 B
----- ×

----- +
2183
PEMBAHASAN

1. Perhatikan bahwa setiap kelipatan 7 hari dari hari Kamis adalah Hari Kamis.
2007 5
 286 
7 7
Karena sisa pembagian 2007 oleh 7 adalah 5, maka 2007 hari setelah Hari Kamis adalah
5 hari setelah Hari Kamis yaitu Hari Selasa.

2. Satu bagian sirup dan 4 bagian air digunakan untuk membuat 60 gelas minuman. Artinya
5 bagian bahan minuman dapat digunakan untuk membuat 60 gelas minuman. Jadi,, 1
bagian sirup dan 5 bagian air sama dengan 6 bagian bahan minuman, sehingga dapat
6
digunakan untuk membuat  60 gelas minuman yaitu 72 gelas minuman.
5

3. Misalkan kedua pecahan itu adalah x dan y maka:


3
x+y=
4
1
x–y=
12
------------- +
5
2x =
6
5
x =
12
3
y= -x
4
3 5
= 
4 12
1
=
3
5 1
Jadi, kedua bilangan itu adalah dan .
12 3

4. Misalkan bilangan 2 digit itu adalah PQ maka


PQ = 10P + Q
QP = 10Q + P
Selisih kedua bilangan itu adalah
PQ – QP = (10P + Q) – (10Q + P)
= 9P – 9Q
= 9(P - Q)
Jadi, selisih kedua bilangan itu merupakan kelipatan 9.

5. Perhatikan gambar di bawah ini!


R

S
c
b
c
a

P Q
b T a

Karena segitiga PTS dan segitiga QRT kongruen, maka  PTS +  QTR = 900 .
Akibatnya  RTS = 1800 - 900
= 900
Jadi, segitiga RST siku-siku.
Kemudian perhatikan bahwa
Luas trapesium PQRS = (2 × Luas segitiga PTS) + (Luas segitiga RST)
1 1 1
 (PS + QR)(PQ) = 2( )(PT)(PS) + (RT)(ST)
2 2 2
1 1 1
 (a + b)(a + b) = 2( )(ab) + (c)(c)
2 2 2
1 1
 (a + b)2 = ab + c2
2 2
 (a + b) = 2ab + c
2 2

 a2 + b2 + 2ab = 2ab + c2
 c2 = a2 + b2

6. Soal ini dapat diselesaikan dengan 3 cara.

Cara I

20 cm
16 cm
S

Q R

Perhatikan bahwa untuk mencari luas segitiga PQR, kita bisa menjadi,kan QR atau PR
sebagai alasnya karena QS dan PR tegaklurus.
1 1
Jadi, (QR)(PQ) = (PR)(QS)  (QR)(PQ) = (PR)(QS)
2 2
 (PQ) PR 2 - PQ2 = (PR)(QS)
 16 400  256 = 20(QS)
 (16)(12)  20 (QS)
 QS = 9,6
Jadi, QS = 9,6 cm

Cara II

Perhatikan bahwa segitiga PQR dan segitiga QRS sebangun maka


PQ PR PQ PR
=  =
QS QR QS PR 2 - PQ2
16 20
 =
QS400 - 256
16 20
 =
QS 12
16 5
 =
QS 3
 QS = 9,6 cm
Jadi, QS = 9,6 cm

Cara III

Perhatikan segitiga PQS dan QRS


Pada segitiga PQS berlaku
QS2 = PQ2 – PS2 … (1)
Pada segitiga QRS berlaku
QS2 = QR2 – RS2 … (2)
Dari persamaan (1) dan (2) diperoleh
PQ2 – PS2 = QR2 – RS2  PQ2 – (20 - RS)2 = (PR2 – PQ2) – RS2
 256 – (RS2 – 40RS + 400) = (400 - 256) – RS2
 - RS2 + 40RS – 144 = 144 – RS2
 40RS = 288
36
 RS =
5
2 2 2
QS = QR – RS
2
 36 
= 144 -  
 5 
= 144 – 51,84
= 92,16
QS = 9,6
Jadi, QS = 9,6 cm

7. Perhatikan gambar berikut ini!


D Q C

P
S

A R B

Segitiga PQS dan QBR kongruen karena ketiga sisi dan sudutnya yang bersesuaian sama.
Pada segitiga PQS berlaku
 QPS +  PQS +  PSQ = 1800   QPS +  PQS + 900 =1800
  QPS +  PQS = 900
  BQR +  PQS = 900
  BQP = 900
Jadi,  BQP = 900.

8. Akan dibuktikan bahwa segitiga PQR sama sisi!


C

E
R F

Q
P

A B
D

Karena  BAC = 60° dan AF adalah garis bagi  BAC, maka  EAP =  DAP = 30°.
Dari segitiga ADP diperoleh
 DAP +  APD +  ADP = 180°  30° +  APD + 90° = 180°
  APD = 60°
Karena garis CD dan AF berpotongan di titik P, maka  RPQ =  APD = 60°
 APD dan  APC saling berpelurus maka:
 APD +  APR = 180°  60° +  APR = 180°
  APR = 120°
Dari segi empat APRE diperoleh
 EAP +  APR +  ERP +  AER = 360°  30° + 120° +  ERP + 90° = 360°
  ERP = 120°
 ERP dan  PRQ saling berpelurus, maka:
 ERP +  PRQ = 180°  120° +  PRQ = 180°
  PRQ = 60°
Dari segitiga PQR diperoleh:
 RPQ +  PRQ +  PQR = 180°  60° + 60° +  PQR = 180°
  PQR = 60°
Karena semua sudut pada segitiga PQR besarnya 60°, maka segitiga PQR sama sisi.

9. Perhatikan gambar di bawah ini!


B

P Q

A C
S R

Karena luas persegi PQRS 1 cm2, maka setiap sisi persegi PQRS berukuran 1 cm.
Pandang segitiga ASP!
Karena segitiga ABC siku-siku dan sama kaki, maka  PAS = 450 sehingga  APS =
45°, akibatnya AS = PS
= 1 cm.
Pandang segitiga BPQ
Karena segitiga BPQ dan segitiga ABC sebangun, maka BP = BQ
Dari segitiga BPQ diperoleh
BP2 + BQ2 = PQ2  2BP2 = 1
1
 BP2 =
2
1
 BP = 2
2
1
Jadi, BP = BQ = 2 cm
2
Luas segitiga ABC = (2 × Luas segitiga ASP) + (Luas persegi PQRS) + (Luas segitiga
BPQ)
1 1
= [ 2( )(AS)(PS) + (PQ)2 + ( )(BP)(BQ) ]
2 2
1
= [ (AS)2 + (PQ)2 + ( )(BP)2 ] cm2
2
1 1
= [ (1)2 + (1)2 + ( 2 )2 ] cm2
2 2
1
=[1+1+ ] cm2
4
1
=2 cm2
4

10. Garis CA melalui titik C(0, -3) dan titik A(3, 0) sehingga garis CA adalah garis y = x
yang digeser 3 satuan ke kanan atau garis y = x yang digeser 3 satuan ke bawah.
Jadi, persamaan garis CA adalah y = x – 3.
y

3 B P

A
x
-3 0 3 6

C -3

Karena garis CA dipotong oleh garis BP yaitu garis y = 3, maka garis CA melalui titik y
= 3.
Akibatnya y = x – 3  3 = x – 3
 x=6
Jadi, koordinat titik P adalah (6, 3)

11. Kecepatan A 20 km/jam sedangkan kecepatan B 60 km/jam , artinya kecepatan B adalah


3 kali lebih cepat dari A atau jarak yang ditempuh B adalah 3 kali lebih besar daripada
jarak yang ditempuh oleh A. Jadi, pada saat mereka berpapasan (bertemu), B sudah
3 3
menempuh jarak dari Kota Q ke Kota P yaitu (160 km) = 120 km sedangkan A
4 4
1 1
baru menempuh jarak jarak dari Kota P ke Kota Q yaitu (160 km) = 40 km.
4 4
40 km
Jadi, A bertemu B setelah  2 jam , yaitu 40 km dari Kota P.
20 km/jam

12. Misalkan Kotak P adalah tempat untuk Calon Presiden, sedangkan Kotak W adalah
tempat untuk Calon Wakil Presiden.

P W
Banyaknya cara untuk mengisi kotak P adalah 5 cara, dimana kelima kandidat boleh
mengisi kotak itu. Karena 1 kandidat telah terpilih dari Kotak P maka Kotak W hanya
dapat diisi oleh 4 kandidat saja akibatnya banyaknya cara untuk mengisi Kotak W adalah
4 cara. Jadi, karena setiap kemungkinan dari kelima kandidat Presiden dapat dilanjutkan
dengan 4 cara untuk memilih 4 kandidat Wakil Presiden maka banyaknya cara untuk
memilih Presiden dan Wakil Presiden dari 5 kandidat adalah 5 x 4 cara yaitu sebanyak 20
cara.
Sebagai ilustrasi misalkan kelima kandidat itu adalah A, B, C, D, dan E maka bagan
untuk semua kemungkinan 2 kandidat yang terpilih adalah
B
A C
D
E
A
C
B
D
o E
o
o
A
B
E
C
D

13. Volume air yang tumpah = Volume kayu yang tenggelam


= 8 cm3

14. Misalakan panjang setiap sisi segitiga ABC adalah 3a.


A

3a 3a

B X P C
a 3/2 a
½a
2a

AP2 = AB2 - BP2


3
= (3a)2 – ( a)2
2
9 2
= 9a2 - a
4
27 2
= a
4
AX2 = AP2 + PX2
27 2 1 2
= a + a
4 4
28 2
= a
4
= 7a2
= 7BX2

15. Perhatikan uraian di bawah ini!


A 7
5 B
x
... Y X
... Z
+
2 1 8 3
Karena digit satuan dari hasil akhir penjumlahan di atas adalah 3, maka nilai X = 3
akibatnya digit B yang mungkin hanyalah 9.
A 7
5 9
x
... Y 3
... Z
+
2 1 8 3
Selanjutnya digit satuan dari hasil perkalian 5 dengan 7 (35) adalah 5, maka haruslah Z =
5
Y+Z=8  Y+5=8
 Y=3
A 7
5 9
x
... 3 3
... 5
+
2 1 8 3
Digit satuan dari hasil perkalian 9 dengan A ditambah digit puluhan dari hasil perkalian 9
dengan 7 (yaitu 6), adalah 3.
Jadi, agar operasi 9 x A + 6 menghasilkan digit satuan 3, maka digit bagi A yang
mungkin adalah 3.
3 7
5 9
x
3 3 3
1 8 5
+
2 1 8 3
Jadi, nilai A + B = 3 + 9
= 12
LATIHAN TAHAP I

TINGKAT PROVINSI

TAHAP I

SOAL ISIAN SINGKAT

1. Jika 44 + 44 + 44 + 44 = 2x, berapakah nilai x?

 1  1  1 
2. Hitunglah nilai dari 1  2 1  2  ... 1  2 
 2  3   2006 

3. Berapakah digit terakhir (angka satuan) pada bilangan 22006?

4. Jika hari ini adalah hari Selasa, hari apakah 1.000 hari yang akan datang?

5. Dalam suatu pertemuan yang dihadiri oleh 25 orang, setiap orang berjabat tangan dengan
setiap orang lainnya tepat 1 kali. Berapa banyaknya jabat tangan yang terJadi,?

6. Diketahui deret huruf ABCDEFGHABCDEFGHABCDEFGH ... . Huruf apakah pada


urutan ke- 222?

7. Diketahui kelompok-kelompok bilangan


(0), (1,2,3), (4, 5, 6, 7, 8), (9, 10, 11, 12, 13, 14, 15), ...
Tentukanlah bilangan yang menempati urutan ke-44 pada kelompok ke-55!

8. Seekor kura-kura mengelilingi sebuah persegi dengan sisi 1 m dan jaraknya selalu tetap
sejauh 1 m dari bagian terluar persegi tersebut. Tentukanlah luas daerah yang dibatasi
oleh lintasan kura-kura tersebut!

9. Saya memikirkan suatu bilangan positif. Kemudian saya mengkuadratkan bilangan itu,
lalu mengalikan hasilnya dengan 4, lalu menguranginya dengan 3, selanjutnya
mengalikan dengan 6 dan terakhir saya menambahnya dengan 80. Saya mendapatkan
bilangan 2006. Berapakah bilangan yang saya pikirkan?
10. Restu selalu berbohong pada hari Senin, Selasa, Rabu dan berkata jujur pada hari-hari
lainnya. Di lain pihak, Imam selalu berbohong pada hari Kamis, Jumat, Sabtu dan berkata
jujur pada hari-hari lainnya. Pada suatu hari terjadi, percakapan berikut:
Restu : “Kemarin saya berbohong”.
Imam : “Saya juga”.
Pada hari apa percakapan tersebut terjadi?

11. Dari bilangan-bilangan berikut, a = 2250, b = 3200, c = 5125, d = 7100, e = 1175. Manakah
bilangan yang paling besar?

12. Diberikan 2100 = 1.267.650.600.228.229.401.496.703.205.376. Tentukan 4 angka pertama


pada bilangan 299 dan 2101!

13. Berapakah jumlah digit-digit bilangan 22005 × 52007?

14. Berapakah sisa pembagian 20062005 oleh 100?

n(n +1)
15. Bilangan segitiga adalah bilangan yang berbentuk , dengan n bilangan bulat
2
positif. Berapa banyak bilangan di antara 100 bilangan segitiga pertama yang memiliki
angka satuan 0?

16. Diketahui bilangan-bilangan asli a, b, dan m. Sisa pembagian a dan b oleh m adalah 1.
Berapakah sisa pembagian ab oleh m?

17. Diketahui 10.125.000 = 23 × 34 × 56. Berapa banyakkah faktor dari 10.125.000?

18. Misalkan M adalah hasil kali semua bilangan prima yang lebih kecil atau sama dengan
101. Berapa banyakkah bilangan prima di antara 100 buah bilangan (M + 2), (M + 3), (M
+ 4), … (M + 101)?

1 1 1
19. Carilah semua bilangan asli x dan y yang memenuhi persamaan - = !
x y 3

20. Bilangan real 0.200620062006… merupakan bilangan rasional sehingga dapat


m
dinyatakan dalam bentuk , dengan m dan n bilangan-bilangan bulat, n  0. Jika
n
dipilih m dan n yang relatif prima, maka berapakah m + n?
(Catatan: Dua bilangan m dan n dikatakan relatif prima jika faktor persekutuan terbesar
dari m dan n adalah 1 )
21. Tuliskanlah 9  80  9  80 dalam bentuk yang paling sederhana!

22. Jika a dan b adalah 2 bilangan positif yang memenuhi ab = ba dan b = 2a. Berapakah nilai
b?

23. Suatu bilangan yang terdiri atas 11 digit, yakni a123456789a, habis dibagi 8. Berapakah
nilai a?

24. Jika x(x - a)(x - b) = x(x - c)(x - d), berapakah x?

25. Misalkan a, b, dan c merupakan bilangan-bilangan real positif yang memenuhi persamaan
a + b2 + 2ac = 29
b + c2 + 2ab = 18
c + a2 + 2bc = 25
Berapakah nilai a + b + c?
KUNCI JAWABAN

1. 10
2007
2.
4012
3. 4
4. Hari Senin
5. 300
6. H
7. 2959
8.   4
9. 9 atau - 9
10. Hari Kamis
11. 6
12. 6338 dan 2535
13. 2006
14. 56
15. 11
16. 1
17. 379
18. 0
19. x = 2 , y = 6
20. 12.005
21. 2 5
22. 4
23. 6
ab - cd
24. 0 atau
(a + b) - (c + d)
25. 8
LATIHAN TAHAP II

TINGKAT PROVINSI

TAHAP II

SOAL URAIAN

1. Misalkan f(x) = x2 + 3x + 2 dan himpunan S = {0, 1, 2, …, 25}. Berapa banyakkah


unsur a di S sehingga f(a) bersisa 0 ketika dibagi 6?

2. Diketahui x dan y adalah bilangan-bilangan berbeda sehingga 2006 + x = y2 dan 2006


+ y = x2. Berapakah nilai xy?

3. Sebuah kereta berangkat dari stasiun A ke stasiun B dengan kecepatan 60 km/jam dan
kembali dari stasiun B ke stasiun A dengan kecepatan 120 km/jam. Berapa kecepatan
rata-rata kereta tersebut?

4. Matematikawan August Morgan menghabiskan seluruh usianya pada tahun 1800-an.


Pada tahun terakhir dalam hidupnya dia mengatakan bahwa: “Dulu Aku berusia x
tahun pada tahun x2”. Pada tahun berapakah August Morgan dilahirkan?

5. Jika Arif memacu sepedanya ke sekolah dengan kecepatan 20 km/jam, maka dia akan
terlambat 5 menit. Jika dia memacu sepedanya dengan kecepatan 30 km/jam, maka
dia akan sampai 5 menit lebih awal. Berapa jauh jarak yang Arif tempuh ke sekolah?

6. Misalakan A = 20062006 , B = 2006 – 2006, C = (- 2006)2006, dan D = (- 2006) – 2006.


Berapakah ABCD?

7. Berapakah nilai dari 1 – 3 + 5 – 7 + 9 – 11 + … - 9999?

612  392
8. Berapakah nilai dari ?
512  492

9. Tentukan semua nilai x yang memenuhi


(200610)(200620)(200630)(200640) = (2006x)x
10. Misalkan Y menyatakan suatu digit. Jika diketahui Y3 × 6528 = 8256 × 3Y,
berapakah Y?

11. Berapakah jumlah digit-digit bilangan 102006 – 2006?

12. Berapa banyaknya bilangan real x yang memenuhi persamaan 32x + 2 - 3x + 3 - 3x + 3 =


0?

13. Misalkan a, b, c, d, dan e adalah bilangan-bilangan bulat positif. Jika diketahui 5a =


4b = 3c = 2d = e, berapakah nilai terkecil untuk a + 2b + 3c + 4d + 5e?

14. Di dalan persegi LMNO terdapat titik P sehingga NOP merupakan segitiga sama sisi.
Berapakah besar sudut PML?

15. Budi berlari lebih cepat dari kecepatan Iwan berjalan kaki. Misalkan Iwan yang lebih
cerdas dari Budi menyelesaikan ujian pada pukul 02:00 dan muai berjalan pulang.
Budi menyelesaikan ujian pada pukul 02:12 dan berlari mengejar Iwan. Pada pukul
berapa Budi tepat menyusul Iwan?
KUNCI JAWABAN

1. 16
2. -2005
3. 80 km/jam
4. 1806
5. 10 km
6. 1
7. -5.000
8. 11
9. 10 atau -10
10. 4
11. 2006
12. 2
13. 3,5
14. 15
15. Pukul 02:18
BAGIAN III TINGKAT NASIONAL

SOAL PEMBAHASAN

Waktu 120 Menit

SOAL URAIAN

1. A adalah suatu himpunan bilangan. Himpunan A memiliki sifat tertutup terhadap


pengurangan, artinya hasil pengurangan dua bilangan di A akan menghasilkan bilangan di A
juga. Jika diketahui dua anggota dari A adalah 4 dan 9, tunjukan bahwa
a. 0  A
b. – 13  A
c. 74  A
d. Selanjutnya, daftarlah semua anggota himpunan A!

2. (2, 0, 4, 1) adalah salah satu solusi dari x1 + x2 + x3 + x4 = 7. Jika semesta pembicaraan pada
persamaan ini adalah himpunan semua bilangan bulat tidak negative, tentukan banyaknya
solusi yang mungkin dari x1 + x2 + x3 + x4 = 7 !

3. Adi adalah karyawan pada salah satu perusaan tekstil yang bertugas menyimpan data tentang
kenaikan produksi selama lima periode. Setelah dicari Adi hanya menemukan empat data
kenaikan, yaitu 4 %, 9 %, 7 %, dan 5 %. Satu data lagi, yaitu data ke-5, tidak ditemukan.
Selidaiki data kenaikan yang ke-5, bila Adi hanya ingat bahwa rata-rata hitung dan median
dari lima data tersebut adalah sama!

4. Tentukan semua pasangan bilangan bulat (x, y) yang memenuhi sistem persamaan berikut :
x(y +1) = y 2 -1
y(x +1) = x 2 -1
5. Perhatikan gambar di bawah ini. ABCD adalah persegi dan E adalah titik sembarang di luar
persegi ABCD. Selidiki apakah berlaku hubungan AE2 + CE2 = BE2 + DE2 pada gambar di
bawah tersebut?
E

D C

A B

6. Sebuah akuarium berbentuk balok dengan ukuran panjang 100 cm, lebar 60 cm dan tinggi 40
cm berisi air cukup banyak. Tino baru saja membersihkan akuarium tersebut, kemudian
menumpahkan sebagian airnya yang dilakukan dengan cara memiringkan akuarium dan
bertumpu pada sisi 60 cm. Ia menghentikan kegiatannya ketika garis permukaan air tepat
berada pada pertengahan dasar akuarium (Lihat gambar). Bila Ia mengembalikan akuarium
ke posisi semula (tegak), berapa ketinggian air yang tersisa sekarang?
40 cm

100 cm

7. Tina diminta menyusun bilangan-bilangan asli lebih besar dari 1 ke dalam tabel yang berisi 5
kolom (a, b, c, d,e) dengan masing-masing baris berisi 4 bilangan saja seperti pada tabel
berikut
a b c d e
2 3 4 5
9 8 7 6
10 11 12 13
17 16 15 14
...

Dengan pola tersebut, pada kolom manakah bilangan 2007 akan diletakkan?

8. Suatu survey rumah tangga di Surabaya memberikan laporan sebagai berikut: 40 % rumah
tangga memiliki 2 anak atau lebih. Di antara keluarga dengan 1 anak, 30 % nya adalah anak
laki-laki. Bila terdapat 10 % keluarga yang tidak memiliki anak, berapa persenkah keluarga
di Surabaya yang memiliki tepat 1 anak perempuan?
9. Diberikan segitiga ABC yang memiliki sudut siku-siku di B, dengan panjang sisi AB = BC =
2 cm. Lengkungan BD dan BE masing-masing adalah busur lingkaran yang berpusat pada C
dan A (lihat gambar). Jika luas lingkaran berjari-jari r adalah  r2, dengan  adalah
konstanta. Hitunglah luas daerah yang diarsir pada gambar tersebut (nyatakan dalam  ).
A

B C

7
10. Di antara 7 buah titik dengan koordinat A(9, 17), B(6, 11), C(3, 5), D(7, 12), E(
, 6), F(5,
2
11), G(5, 9), lima di antaranya terletak pada garis lurus. Dua titik manakah yang tidak
terletak pada garis tersebut?

PEMBAHASAN

1. a. Karena 4  A ,
Maka 4 – 4 = 0  A
b. Karena 9 dan 4 anggota A
Maka 9 – 4 = 5  A
4–9=-5 A
4–5=-1 A
- 5 – 5 = - 10  A
- 10 – 4 = - 14  A
- 14 – (- 1) = - 13  A
c. 9  A dan (– 1)  A
Jika 9 dikurangi oleh ( – 1) terus menerus sampai 65 kali pengurangan, maka hasilnya
adalah 74.
Jadi, 74  A
d. Dari bagian (a) dan (b) diperoleh bahwa 0  A dan (– 1)  A , kemudian dari
pengurangan oleh 4 diperoleh bahwa 1  A.
Jika 1 dikurangkan oleh (– 1) terus menerus, masing-masing akan menghasilkan 2, 3,
4, …
Jadi, {2, 3, 4, …}  A.
Kemudian jika (– 1) dikurangkan oleh 1 terus menerus masing-masing akan menghasil
kan -2, - 3, - 4, …
Jadi, {… , - 4, - 3, -2}  A
Karena {- 1, 0, 1}  A, {2, 3, 4, …}  A dan {… , - 4, - 3, -2}  A
Maka A = {… , - 4, - 3, -2}  {- 1, 0, 1}  {2, 3, 4, …}
= {… , - 2, - 1, 0, 1, 2, …}
Jadi, seluruh anggota A adalah himpunan semua bilangan bulat.

2. Akan dikelompokkan semua bilangan mulai dari yang berdigit 1 sampai yang berdigit 4
tetapi jumlah digit-digitnya 7.
 Berdigit 1
0007
---------
1
Hanya terdapat 1 kemungkinan
 Berdigit 2

0016
0025

0061
0070
---------
7
Terdapat 7 kemungkinan
 Berdigit 3

0106 0205 0700


0115 0214

0151 0241
0160 0250
--------- --------- --------
7 6 1
Banyaknya kemungkinan adalah (7 + 6 + 5 + … + 1) yang merupakan jumlah
deret aritmatika (Sn) dengan n = 7, U1 = 7, Un = 1, b = - 1
n
Sn = ( U1 + Un)
2
7
= (7 + 1)
2
7
= (8)
2
= 28

 Berdigit 4
1006 1105 1600
1015 1114

1051 1141
1060 1150
--------- --------- ---------
7 6 1
2005 2104 2500
2014 2113

2041 2131
2050 2140
--------- --------- ---------
6 5 1

7000
---------
1
Banyaknya kemungkinan adalah
(7 + 6 + … + 1) + (6 + 5 + … + 1) + … + 1 = 28 + 21 + 15 + 10 + 6 + 3 + 1
= 84
Jadi, banyaknya solusi yang mungkin adalah (1 + 7 + 28 + 84) = 120

3. Misalkan X adalah rata-rata kelima data tersebut dan y adalah data kenaikan ke-5 yang
hilang.
4+9+7+5+ y
Maka X =
5
25 + y
=
5
y
=5+
5
Karena rata-rata kelima data tersebut sama dengan mediannya
y
Maka Median = 5 + .
5
Karena banyaknya data adalah 5, maka median adalah data yang ke-3 setelah diurutkan.
Kelima data yang telah diurutkan itu adalah
y
4, 5, 5 + , 7, 9
5
y y
Akibatnya 5 ≤ 5 + ≤ 7  0 ≤ ≤ 2
5 5
 0  y  10
Jadi, kenaikan produksi pada periode ke-5 berkisar antara 0 % sampai 10 %.

4. Akan dicari semua pasangan bilangan bulat (x, y) yang memenuhi sistem persamaan (1)
dan (2).
x(y + 1) = y2 – 1 … (1)
y(x + 1) = x2 – 1 … (2)
Dari persamaan (1) diperoleh:
x(y + 1) = y2 – 1  x(y + 1) = (y +1)(y – 1)
Agar kedua ruas pada persamaan terakhir sama, maka y = - 1 atau jika y  - 1
mengharuskan x = y – 1
 Jika y = - 1
Maka pada uraian persamaan (1) tidak berlaku hukum pencoretan (y + 1) pada
kedua ruas karena nilai (y + 1) adalah 0. Oleh karena itu nilai y = - 1
disubstitusikan ke persamaan (2)
y(x + 1) = x2 – 1  (- 1)(x + 1) = x2 – 1
 - x – 1 = x2 – 1
 x2 + x = 0
 x(x + 1) = 0
 x = 0 atau x = - 1
Jadi, pasangan bilangan bulatnya adalah (- 1, - 1) dan (0, - 1)
 Jika y  - 1
x = y – 1  y = x + 1 … (3)
Kemudian substitusikan persamaan (3) ke persamaan (2)
y(x + 1) = x2 – 1  (x + 1) (x + 1) = x2 – 1
 x2 + 2x + 1 = x2 – 1
 2x = - 2
 x=-1
Substitusikan nilai x = - 1 ke persamaan (3)
y=x+1  y=-1+1
=0
Jadi, pasangan bilangan bulatnya adalah (- 1, 0).
Jadi, seluruh pasangan bilangan bulat (x, y) yang memenuhi persamaan (1) dan (2) adalah
(- 1, - 1), (- 1, 0) dan (0, - 1).
5. Perhatikan bahwa titik E terletak di sembarang tempat di Luar persegi ABCD. Karena
ABCD persegi, maka kita cukup menempatkan titik E di titik sebelah luar salah satu sisi
persegi, misalnya di sebelah luar sisi DC .
E

D C
F

A G B

Dari segitiga DEF diperoleh


EF2 = DE2 – DF2 … (1)
Dari segitiga CEF diperoleh
EF2 = CE2 – CF2 … (2)
Dari persamaan (1) dan (2) diperoleh
DE2 – DF2 = CE2 – CF2  CF2 - DF2 = CE2 – DE2 … (3)
Dari segitiga AEG diperoleh
EG2 = AE2 – AG2 … (4)
Dari segitiga BEG diperoleh
EG2 = BE2 – BG2 … (5)
Dari persamaan (4) dan (5) diperoleh
AE2 – AG2 = BE2 – BG2  AE2 – DF2 = BE2 – CF2
 CF2 - DF2 = BE2 – AE2 … (6)
Dari persamaan (3) dan (6) diperoleh
CE2 – DE2 = BE2 – AE2  AE2 + CE2 = BE2 + DE2
Jadi, terbukti bahwa pada persegi ABCD dengan E sebarang titik di luar persegi tersebut,
akan berlaku hubungan AE2 + CE2 = BE2 + DE2.

6. Perhatikan bahwa volume air setelah akuarium dikembalikan ke posisi semula sama
dengan volume air pada saat akuarium dalam posisi miring. Volume air pada saat
akuarium miring adalah volume sebuah prisma yang tingginya sama dengan lebar
akuarium, sedangkan alas prisma berupa segitiga siku-siku yang panjang alasnya sama
dengan setengah dari panjang akuarium dan tingginya sama dengan tiggi akuarium.
Misalkan: P = Panjang akuarium
= 100 cm
L = Lebar akuarium
= 60 cm
T1 = tinggi air pada akuarium Semula
= 40 cm
T2 = tinggi air setelah akuarium dikembalikan ke posisi semula
V1 = Volume air setelah akuarium dikembalikan ke posisi semula
V2 = Volume prisma segitiga siku-siku
V1 = V2
1 1
 P × L × T2 = × ( P) × T1 × L (Kedua ruas dibagi dengan PL)
2 2
1
 T2 = T1
4
1
 T2 = (40 cm)
4
= 10 cm
Jadi, tinggi air setelah akuarium dikembalikan ke posisi semula adalah 10 cm.

7. Perhatikan tabel pada soal!


Dari semua kolom, hanya kolom c yang mempunyai keteraturan pada setiap barisnya.
Setiap baris pada kolom c merupakan kelipatan 4. Perhatikan bahwa khusus untuk
bilangan kelipatan 4 yang juga merupakan kelipatan 8, pergerakan bilangan selanjutnya
berarah ke sebelah kiri, sedangkan yang lainnya berarah ke sebelah kanan.
Selanjutnya akan dicari suatu bilangan kelipatan 4 terbesar yang lebih kecil atau sama
dengan 2007 yaitu 2004. Karena 2004 bukan kelipatan 8, maka 2 bilangan asli
selanjutnya setelah 2004 yaitu 2005 dan 2006 bergerak ke arah kanan sehingga masing-
masing menempati kolom d dan kolom e. Jadi, bilangan 2007 menempati kolom d.

8. Misalkan A = Persentase banyaknya rumah tangga yang memiliki 2 anak atau lebih
= 40 %
B = Persentase banyaknya rumah tangga yang memiliki tepat 1 anak
C = Persentase banyaknya rumah tangga yang tidak memiliki anak
= 10 %
T = Persentase banyaknya rumah tangga total di Surabaya
=A+B+C
= B + 50 %
= 100 %
Akibatnya B = 50 %
Kemudian karena 30 % dari B memiliki anak perempuan, maka persentase rumah tangga
yang memiliki tepat satu anak perempuan adalah 70 % dari B yaitu 70 % × 50 % = 35 %.
Jadi, persentase rumah tangga yang memiliki tepat satu anak di Surabaya adalah 35 %.

9. Perhatikan gambar di bawah ini!


Pada segitiga ABC, BF tegaklurus AC dan AG tegaklurus BE.
A

F
20 cm
E
G

B 20 cm C

Perhatikan juring ABE !


Karena AB = BC, maka  BAD =  BCE = 450. Akibatnya juring ABE = juring CBD .
450
Luas juring ABE = 0
πr 2
360
1
= π(AB) 2
8
1
= π  20 cm 
2

8
= 50 π cm2 .
Dari segitiga ABC diperoleh
AC2 = AB2 + BC2
= (202 + 202) cm
= (400 + 400) cm
= 800 cm
AC = 800 cm
= 20 2 cm
1
AF = AC
2
1
= ( 20 2 ) cm
2
= 10 2 cm
Karena BF tegak-lurus dengan AC maka pada segitiga ABF berlaku  ABF =  BAF =
450. Akibatnya BF = AF
= 10 2 cm.
Perhatikan bahwa CD = AB
=r
= 20 cm.
DF = CD – CF
= CD – AF
= (20 - 10 2 ) cm
= 10(2 - 2 ) cm
1
Luas segitiga BDE = (DE)(BF)
2
1
= (2 × DF)(BF)
2
= (DF)(BF)
= 10(2 - 2 )(10 2 ) cm2
= 100 2 (2 - 2 ) cm2
= [200 2 - 200] cm2
= 200( 2 - 1) cm2
Dari segitiga BEF diperoleh
BE2 = BF2 + EF2
= [(10 2 )2 + (20 - 10 2 )2] cm
= [200 + 400 + 200 - 400 2 ] cm
= [800 - 400 2 ] cm
= 400(2 - 2)
BE = 
400 2  2 cm 
= 20 2  2 cm
1
BG = BE
2
1
= (20 2  2 ) cm
2
= 10 2  2 cm
Kemudian dari segitiga ABG diperoleh
AG2 = AB2 – BG2
= (20)2 – (10 2  2 )2 cm
= [400 – 100(2 - 2 )] cm
= (400 – 200 + 100 2 ) cm
= (200 + 100 2 ) cm
= 100(2 + 2 ) cm


AG = 100 2  2 cm 
= 10 2  2 cm
1
Luas segitiga ABE = (BE)(AG)
2
1
= (20 2  2 cm)( 10 2  2 cm)
2
= 100  2  2  2  2  cm 2

= 100 4  2 cm2
= 100 2 cm2
Luas tembereng BE = (Luas juring ABE ) – (Luas segitiga ABE)
= (50  - 100 2 ) cm2
= 50(  - 2 2 ) cm2
Luas daerah diarsir pada segitiga ABC = 2 × (Luas diarsir pada juring)
= 2 × [(Luas juring ABE ) – 2 × (Luas tembereng BE ) - (Luas segitiga BDE)]
= 2[50  - (2)50(  - 2 2 ) - 200( 2 - 1)] cm2

= 2[50  - 100  + 200 2 - 200 2 + 200] cm2

= 2[200 - 50  ] cm2

= 100(4 -  ) cm2

Jadi, luas daerah yang diarsir adalah 100(4 -  ) cm2.

Catatan :

Untuk mencari luas sebarang segitiga yang diketahui ketiga sisinya dapat digunakan juga

rumus

L= s(s - a)(s - b)(s - c)

Dimana L = Luas segitiga

a, b, c = Panjang sisi-sisinya

s = Setengah keliling

a +b+c
=
2
10. Pertama-tama akan dipilih sembarang dua titik dari 7 titik yang diketahui untuk dibuat
suatu persamaan garis. Selanjutnya substitusikan ketujuh titik tersebut ke persamaan garis
yang telah dibuat sehingga tepat 5 titik memenuhi persamaan garis. Apabila tidak tepat 5
titik memenuhi persamaan garis artinya kita telah salah memilih 2 titik sebarang.
Misalkan 2 titik yang dipilih adalah titik A = (x1, y1) = (9, 17) dan B = (x1, y1) = (6, 11).
Persamaan garis lurus yang melalui A dan B adalah
y - y1 x - x1 y -17 x - 9
=  =
y2 - y1 x 2 - x1 11-17 6 - 9
y -17 x - 9
 =
-6 -3
y -17
 = x -9
2
 y -17 = 2x -18
 2x – y – 1 = 0
Titik A, B, C, E dan G memenuhi persamaan terakhir di atas sedangkan titik D dan F
tidak memenuhi.
Jadi, 2 titik yang tidak terletak pada garis yang ditempati kelima titik lainnya adalah titik
D(7, 12) dan titik F(5, 10).
LATIHAN

BIDANG
MATEMATIKA

SOAL URAIAN

1. Bentuk paling sederhana dari 2  2 4  12 = …

2. Ada berapa banyak bilangan bulat positif 3 angka, dimana angka ketiga merupakan
jumlah kedua angka di depannya?

3. Diketahui ada bilangan 5 digit dengan ciri sebagai berikut: digit puluhan adalah dua kali
lipat digit ribuan, serta apabila digit ratusan dan satuan dipertukarkan maka nilai bilangan
tersebut tidak berubah. Terdapat berapa banyakkah bilangan tersebut?

4. Tiga sahabat Ani, Ina, dan Nia sedang membicarakan uang saku bulanan yang mereka
terima dari orang tua mereka masing-masing. Ani merasa heran, karena uang sakunya
hanya setengah dari uang saku Ina, bahkan Nia mendapatkan lebih banyak lagi, karena
selisihnya dengan uang saku Ina adalah dua kali lipat dari uang saku Ani. Diketahui
jumlah uang saku ketiganya adalah Rp. 350.000. Berapa besarnya uang saku Ani?

1 1 1
5. Tentukan semua pasangan bilangan asli (x,y) sehingga + = !
x y 3

6. Diketahui x = 0, 1234567891011 … 998999. Perhatikan bahwa angka-angka di belakang


tanda koma adalah bilangan-bilangan asli kurang dari 1.000 yang disusun berurutan .
Berikan angka ke-2007 di belakang tanda koma!

7. Terdapat berapa banyakkah bilangan bulat positif tak lebih dari 2007 yang merupakan
kelipatan 3 atau 4 tetapi bukan kelipatan 5?

8. Diberikan pola bilangan sebagai berikut


1, 2, 4, 7, 11, 16, …
Tentukan bilangan ke-2007 pada barisan tersebut!
9. Jika diketahui n adalah kuadrat suatu bilangan asli, maka tentukan bilangan kuadrat
berikutnya yang dinyatakan dalam n!

10. Lima titik A, B, C, D dan E terletak berurutan pada suatu garis lurus. Diketahui bahwa
jarak A ke E adalah 20 cm, A ke D 15 cm, B ke E 10 cm, dan titik C terletak di tengah-
tengah antara B dan D. Berapakah jarak B ke D?
KUNCI JAWABAN

1. 1+ 3
2. 45
3. 450
4. Rp. 50.000
5. {(2, 6)}
6. 5
7. 804
8. 2.013.022
9. n + n 2 +1

10. 2,5 cm
DAFTAR PUSTAKA

Frank C. , 2000, Australian Mathematics Olympiad for Junior High School, Sydney Press.

Gardiner A. ,1997, Discovering mathematics, Clarendon press – Oxford.

_ , 2001, Text of Canadian Mathematics Olympiad for Junior High school.

_ , 2003, Text of Canadian Mathematics Olympiad for junior High School.

_ , 2015, Naskah Olimpiade Matematika Tingkat Kabupaten / Kota, Provinsi dan Nasional, Depdiknas.
2003 Fryer Contest (Grade 9)
Wednesday, April 16, 2003
1. (a) The marks of 32 mathematics students on Test 1 Marks on Test 1
are all multiples of 10 and are shown on the bar
10
graph. What was the average (mean) mark of the
32 students in the class? 9
8

Number of Students
7
6
5
4
3
2
1

0 10 20 30 40 50 60 70 80 90 100
Marks (out of 100)
(b) After his first 6 tests, Paul has an average of 86. What will his average be if he scores 100 on his next test?
(c) Later in the year, Mary realizes that she needs a mark of 100 on the next test in order to achieve an average
of 90 for all her tests. However, if she gets a mark of 70 on the next test, her average will be 87. After she
writes the next test, how many tests will she have written?

2. In a game, Xavier and Yolanda take turns calling out whole numbers. The first number called must be a whole
number between and including 1 and 9. Each number called after the first must be a whole number which is 1
to 10 greater than the previous number called.
(a) The first time the game is played, the person who calls the number 15 is the winner. Explain why Xavier
has a winning strategy if he goes first and calls 4.
(b) The second time the game is played, the person who calls the number 50 is the winner. If Xavier goes first,
how does he guarantee that he will win?

3. In the diagram, ABCD is a square and the coordinates of A and D y


are as shown.
(a) The point P has coordinates (10, 0 ) . Show that the area of D(1, 8) C
triangle PCB is 10.
(b) Point E( a, 0) is on the x-axis such that triangle CBE lies
entirely outside square ABCD. If the area of the triangle is
equal to the area of the square, what is the value of a? A(1, 4) B
(c) Show that there is no point F on the x-axis for which the area
of triangle ABF is equal to the area of square ABCD.
x
P(10, 0)
4. For the set of numbers {1, 10, 100} we can obtain 7 distinct numbers as totals of one or more elements of the set.
These totals are 1, 10, 100, 1 + 10 = 11, 1 + 100 = 101, 10 + 100 = 110 , and 1 + 10 + 100 = 111 . The “power-sum”
of this set is the sum of these totals, in this case, 444.
(a) How many distinct numbers may be obtained as a sum of one or more different numbers from the set
{1, 10, 100, 1000} ? Calculate the power-sum for this set.
(b) Determine the power-sum of the set {1, 10, 100, 1000, 10 000, 100 000, 1 000 000} . over ...
Extensions (Attempt these only when you have completed as much as possible of the four main
problems.)

Extension to Problem 1:
Mary’s teacher records the final marks of the 32 students. The teacher calculates that, for the entire class, the median
mark is 80. The teacher also calculates that the difference between the highest and lowest marks is 40 and calculates
that the average mark for the entire class is 58. Show that the teacher has made a calculation error.

Extension to Problem 2:
In the game described in (b), the target number was 50. For what different values of the target number is it guaranteed
that Yolanda will have a winning strategy if Xavier goes first?

Extension to Problem 3:
G is a point on the line passing through the points M ( 0, 8 ) and N ( 3, 10 ) such that ∆ DCG lies entirely outside the
square. If the area of ∆ DCG is equal to the area of the square, determine the coordinates of G.

Extension to Problem 4:
Consider the set {1, 2, 3, 6, 12, 24, 48, 96}. How many different totals are now possible if a total is defined as the
sum of 1 or more elements of a set?
Canadian
Mathematics
Competition
An activity of The Centre for Education
in Mathematics and Computing,
University of Waterloo, Waterloo, Ontario

2003 Solutions
Fryer Contest (Grade 9)

© 2003 Waterloo Mathematics Foundation


2003 Fryer Solutions 2

1. (a) Solution 1
The average (mean) is equal to the sum of all of the marks, divided by the total number of
marks.
Since we know already that there are 32 students (we can check this by looking at the
graph), then the average is
1(10) + 2( 30) + 2( 40) + 1(50) + 4 (60) + 6( 70) + 9(80) + 4 (90) + 3(100) 2240
= = 70
32 32
Therefore, the average mark was 70.

Solution 2
The average (mean) is equal to the sum of all of the marks, divided by the total number of
marks.
Using the bar graph, we can list out all of the marks:
10, 30, 30, 40, 40, 50, 60, 60, 60, 60, 70, 70, 70, 70, 70, 70, 80, 80, 80, 80, 80, 80, 80, 80,
80, 90, 90, 90, 90, 100, 100, 100.
Adding these up using a calculator and dividing by 32, we find that the average mark is
70.

(b) After his first 6 tests, since Paul’s average is 86, then has gotten a total of 6(86) = 516
marks.
After getting 100 on his seventh test, Paul has gotten a total of 516 + 100 = 616 marks, so
616
his new average is = 88 .
7

(c) Solution 1
If Mary gets 100, her average becomes 90.
If Mary gets 70, her average becomes 87.
So a difference of 30 marks on the test gives a difference of 3 marks in the average.
Since her average is her total number of marks divided by her total number of tests, and a
difference of 30 in the total number of marks makes a difference of 3 in her average, then
30
she will have written = 10 tests.
3

Solution 2
Suppose that after the next test, Mary has written n tests.
If her average after getting 100 on the next test is 90, then Mary has earned 90n marks in
total after the first n tests, and so 90 n − 100 before she writes the nth test.
If her average after getting 70 on the next test is 87, then Mary has gotten 87n marks in
total after the nth test, and so she will have earned 87 n − 70 marks before the next test.
Therefore, since the number of marks before her next test is the same in either case,
2003 Fryer Solutions 3

87 n − 70 = 90 n − 100
30 = 3n
n = 10
So Mary will have written 10 tests.

Extension
We start by using the given information to try to figure out some more things about the
marks of the 32 students.
Since the median mark is the “middle mark” in a list of marks which is increasing, then
there at least 16 students who have marks that are at least 80.
Since the difference between the highest and lowest marks is 40, and there are students
who got at least 80, then the lowest mark in the class cannot be lower than 40.
Since the average mark in the class is 58, then the total number of marks is
32(58) = 1856 .
So what does this tell us?
Since at least 16 students got at least 80, then this accounts for at least 1280 marks,
leaving 1856 − 1280 = 576 marks for the remaining 16 students.
But the lowest possible mark in the class was 40, so these remaining 16 students got at
least 40 each, and so got at least 16( 40) = 640 marks in total!
So we have an inconsistency in the data.
Thus, the teacher made a calculation error.

(There is a variety of different ways of reaching this same conclusion. As before, we


know that 16 students will have a mark of at least 80, which accounts for 1280 marks.
By the same reasoning, the other 16 students would account for the other 576 marks. The
average for these 16 students is thus about 34, which implies that at least one of these
lower students must have a mark of 34 or lower. This now contradicts the statement that
the range is 40 since 80 − 34 > 40 .)

2. (a) Solution 1
If Xavier goes first and calls 4, then on her turn Yolanda can call any number from 5 to
14, since her number has to be from 1 to 10 greater than Xavier’s.
But if Yolanda calls a number from 5 to 14, then Xavier can call 15 on his next turn,
since 15 is from 1 to 10 bigger than any of the possible numbers that Yolanda can call.
So Xavier can call 15 on his second turn no matter what Yolanda calls, and is thus always
guaranteed to win.
2003 Fryer Solutions 4

Solution 2
If Xavier goes first and calls 4, then Yolanda will call a number of the form 4 + n where
n is a whole number between 1 and 10.
On his second turn, Xavier can call 15 (and thus win) if the difference between 15 and
4 + n is between 1 and 10. But 15 − ( 4 + n ) = 11 − n and since n is between 1 and 10,
then 11− n is also between 1 and 10, so Xavier can call 15.
Therefore, Xavier’s winning strategy is to call 15 on his second turn.

(b) In (a), we saw that if Xavier calls 4, then he can guarantee that he can call 15.
Using the same argument, shifting all of the numbers up, to guarantee that he can call 50,
he should call 39 on his previous turn.
(In this case, Yolanda can call any whole number from 40 to 49, and in any of these cases
Xavier can call 50, since 50 is no more than 10 greater than any of these numbers.)
In a similar way, to guarantee that he can call 39, he should call 28 on his previous turn,
which he can do for the same reasons as above.
To guarantee that he can call 28, he should call 17 on his previous turn.
To guarantee that he can call 17, he should call 6 on his previous turn, which could be his
first turn.
Therefore, Xavier’s winning strategy is to call 6 on his first turn, 17 on his second turn,
28 on his third turn, 39 on his fourth turn, and 50 on his last turn.
At each step, we are using the fact that Xavier can guarantee that his number on one turn
is 11 greater than his number on his previous turn. This is because Yolanda adds 1, 2, 3,
4, 5, 6, 7, 8, 9, or 10 to his previous number, and he can then correspondingly add 10, 9,
8, 7, 6, 5, 4, 3, 2, or 1 to her number, for a total of 11 in each case.

Extension
In (b), we discovered that Xavier can always guarantee that the difference between his
numbers on two successive turns is 11.
In fact, Yolanda can do the same thing, using exactly the same strategy as Xavier did.
If the target number is between 1 and 9, then Xavier will win on his very first turn by
calling that number.
If the target number is then 11 greater than a number between 1 and 9, Xavier will win as
in (b). Thus, Xavier wins for 12 through 20.
What about 10 and 11? In each of these cases, Yolanda can win by choosing 10 or 11 on
her first turn, which she can do for any initial choice of Xavier’s, since he chooses a
number between 1 and 9.
Therefore, Yolanda will also win for 21 and 22, and so also for 32 and 33, and so on.
Since either Yolanda or Xavier can repeat their strategy as many times as they want, then
Xavier can ensure that he wins if the target number is a multiple of 11 more than one of 1
through 9.
2003 Fryer Solutions 5

Similarly, Yolanda can ensure that she wins if the target number is a multiple of 11 more
than 10 or 11, ie. if the target number is a multiple of 11, or 1 less than a multiple of 11.

3. (a) Solution 1
Since ABCD is a square and AD has side length 4, then each of the sides of ABCD has
length 4. We can also conclude that B has coordinates (5, 4 ) and C has coordinates (5, 8) .
(Since AD is parallel to the y-axis, then AB is parallel to the x-axis.)

If we turn ∆PBC on its side, then its we see y


that its base is BC which has length 4. Also
the height of the triangle is the vertical D(1, 8) C(5, 8)
distance from the line BC to P, which is 5.
(We can see this by extending the line CB to a 4
B(5, 4)
point X on the x-axis. Then X has coordinates
(5, 0) and PX, which has length 5, is A(1, 4)
4
perpendicular to CB.)
Therefore, the area of ∆PBC is x
X(5, 0) 5 P(10, 0)
1
2
bh = 12 ( 4 )(5) = 10 .

Solution 2
Since ABCD is a square and AD has side length 4, then all of the sides of ABCD have
length 4. We can also conclude that B has coordinates (5, 4 ) and C has coordinates (5, 8) .
(Since AD is parallel to the y-axis, then AB is parallel to the x-axis.)

Extend CB down to a point X on the x-axis. y


Point X has coordinates (5, 0) .
Then the area of ∆PBC is the difference D(1, 8) C(5, 8)
between the areas of ∆PCX and ∆PBX .
∆PCX has base PX of length 5 and height CX 4
B(5, 4)
of length 8.
∆PBX has base PX of length 5 and height BX A(1, 4)
4
of length 4.
x
Therefore, the area of ∆PBC is X(5, 0) 5 P(10, 0)
1
2
(4)(10) − 12 (4)(5) = 10
as required.

(b) Solution 1
Since triangle CBE lies entirely outside square ABCD, then the point E must be “to the
right” of the square, ie. a must be at least 5.
2003 Fryer Solutions 6

Also, we need to know the area of the square. Since the side length of the square is 4, its
area is 16.

Thus if we turn ∆CBE on its side, then its we y


see that its base is BC which has length 4.
Also the height of the triangle is the vertical D(1, 8) C(5, 8)
distance from the line BC to E, which is a − 5
since E has coordinates ( a,0) and BC is part 4
of the line x = 5 .
A(1, 4) B(5, 4)
Therefore, since the area of ∆CBE is equal to
the area of the square,
1
2
(4)(a − 5) = 16 E(a, 0)
x

2 a − 10 = 16
2 a = 26
a = 13
Thus, a = 13.
(It is easy to verify that if a = 13, then the height of the triangle is 8 and its base is 4,
giving an area of 16.)

Solution 2
Since triangle CBE lies entirely outside square ABCD, then the point E must be “to the
right” of the square, ie. a must be at least 5.
Also, we need to know the area of the square. Since the side length of the square is 4, its
area is 16.

Extend CB down to a point X on the x-axis. y


Point X has coordinates (5, 0) .
Then the area of ∆CBE is the difference D(1, 8) C(5, 8)
between the areas of ∆CXE and ∆BXE .
∆CXE has base EX of length a − 5 and 4
B(5, 4)
height CX of length 8.
∆BXE has base EX of length a − 5 and height A(1, 4)
4
BX of length 4.
x
Therefore, since the area of ∆CBE is equal to X(5, 0) a–5 E(a, 0)
the area of the square,
1
2
(a − 5)(8) − 12 (a − 5)(4) = 16
4 ( a − 5) − 2( a − 5) = 16
a−5= 8
a = 13
2003 Fryer Solutions 7

Thus, a = 13, as required.

(c) Suppose that F has coordinates (b,0) . y


Then triangle ABF has base AB of length 4.
The height of triangle ABF is the vertical distance from F to D(1, 8) C(5, 8)
the line AB, which is always 4, no matter where F is.
Thus, the area of triangle ABF is always 12 bh = 12 ( 4 )( 4 ) = 8 ,
A(1, 4)
which is not equal to the area of the square. B(5, 4)

x
F(b, 0)

Extension
Since triangle DCG lies entirely outside the square, then G is “above” the line through D
and C, ie. the y-coordinate of G is at least 8.
Since the area of triangle DCG is equal to the area of the square, then the area of triangle
DCG is 16.
Now triangle DCB has base DC, which has length y
4, so 2 bh = 2 ( 4 ) h = 16 or h = 8 .
1 1
G
Since the height of triangle DCG is 8, then G has
y-coordinate 16, since both D and C have
N(3, 10)
y-coordinate 8.
So we must find the point on the line through M(0, 8)
M (0, 8) and N ( 3,10) which has y-coordinate 16.
D(1, 8) C(5, 8)
To get from M to N, we go 3 to the right and up 2.
To get from N to G, we go up 6, so we must go 9
to the right. A(1, 4) B(5, 4)
Therefore, G has coordinates G(12,16) .
x

4. (a) The best approach here is to list the numbers directly. The possible totals are, from
smallest to largest:
1, 10, 11, 100, 101, 110, 111, 1000, 1001, 1010, 1011, 1100, 1101, 1110, 1111
There 15 possible totals, and their sum (that is, the power-sum) is 8888.

(b) Solution 1
First, we consider the numbers that are sums of 1 or more of the numbers from
{110
, ,100,1000} . In (a), we saw that the sum of these numbers is 8888.
2003 Fryer Solutions 8

What happens when we consider numbers that are sums of 1 or more of the numbers
from {110 , ,100,1000,10 000} ? When we do this, we obtain all 15 of the numbers from the
previous paragraph, we obtain the 15 numbers obtained by adding 10 000 to all of the
numbers from the previous paragraph, and also the number 10 000. (Either our sum does
not include 10 000 as a term, or it does; if it doesn’t, it must be one of the numbers from
(a); if it does, it could be 10 000 on its own, or it could be 10 000 plus one of the numbers
from (a).)
Therefore, we have 15 + 15 + 1 = 31 numbers in total, whose sum is
[ ]
8888 + 8888 + 15(10 000 ) + 10 000 = 2( 8888 ) + 160 000 = 2( 88 888 ) = 177 776

What happens when we consider the numbers that are sums of 1 or more of the numbers
from {110
, ,100,1000,10 000,100 000} ? When we do this, we obtain all 31 of the numbers
from (a), we obtain the 31 numbers obtained by adding 100 000 to all of the numbers
from (a), and also the number 100 000.
Therefore, we have 31 + 31 + 1 = 63 numbers in total, whose sum is
[ ]
177 776 + 177 776 + 31(100 000 ) + 100 000 = 2(177 776 ) + 3 200 000
= 3 555 552

What happens when we add 1 000 000 to the set? We then obtain, as before,
63 + 63 + 1 = 127 numbers in total, whose sum is
[ ]
3 555 552 + 3 555 552 + 63(1 000 000 ) + 1 000 000 = 2( 3 555 552 ) + 64 000 000
= 71111104
Therefore, the power sum is 71 111 104.

Solution 2
There are seven numbers in the given set. When we are considering sums of one or more
numbers from the set, each of the seven numbers in the set is either part of the sum, or
not part of the sum. So there are two choices (“in” or “out”) for each of the 7 elements.

So we can proceed by first choosing the elements we want to add up, and then adding
them up. Since for each of the two possibilities for the “1” (ie. chosen or not chosen),
there are two possibilities for the “10”, and there are two possibilities for the “100”, and
so on. In total, there will be 2 × 2 × 2 × 2 × 2 × 2 × 2 = 2 7 = 128 ways of choosing
elements. Notice that this includes the possibility of choosing no elements at all (since
we could choose not to select each of the seven elements).

So there are 128 possible sums (including the sum which doesn’t add up any numbers at
all!).
2003 Fryer Solutions 9

In how many of these sums is the “1” chosen? If the “1” is chosen, then there are still 2
possibilities for each of the remaining six elements (either chosen or not chosen), so there
are 2 6 = 64 sums with the 1 included, so the 1 contributes 64 to the power-sum.
In how many of these sums is the “10” chosen? Using exactly the same reasoning, there
are 64 sums which include the 10, so the 10 contributes 640 to the power-sum.

Extending this reasoning, each of the 7 elements will contribute to 64 of the sums. (Note
that including the “empty” sum doesn’t affect the power-sum.)
Therefore, the power sum is
64 (1 + 10 + 100 + 1000 + 10000 + 100000 + 1000000)
= 64 (1111111)
= 71111104

Extension
We start looking at small numbers to see if we can see a pattern.
Using the numbers 1, 2, 3, we can form the sums
1, 2, 3, 1 + 3 = 4 , 1 + 3 = 4 , 2 + 3 = 5 , 1 + 2 + 3 = 6
If we include the 6, we can obtain all of these sums, as well as 6 plus each of these sums.
In other words, we obtain each of the numbers 1 through 12 as totals.
Then including the 12, we can obtain 13 through 24, so we now have each of 1 to 24 as
totals.
Including the 24, we obtain all numbers up to 48.
Including the 48, we obtain all numbers up to 96.
Including the 96, we obtain all numbers up to 192.
Therefore, there are 192 different totals possible.
(We can check as well that the sum of the elements in the original set is 192.)
Canadian
Mathematics
Competition
An activity of The Centre for Education
in Mathematics and Computing,
University of Waterloo, Waterloo, Ontario

Pascal Contest (Grade 9)


Wednesday, February 24, 1999

C.M.C. Sponsors: C.M.C. Supporters: C.M.C. Contributors:

The Great-West
Life Assurance
Company
IBM
Canada Ltd.
Northern Telecom
(Nortel)

Manulife
Financial
Canadian Institute
of Actuaries
Equitable Life
of Canada

Sybase
Inc. (Waterloo)
Chartered Accountants

Time: 1 hour © 1999 Waterloo Mathematics Foundation


Calculators are permitted, providing they are non-programmable and without graphic displays.
Instructions
1. Do not open the contest booklet until you are told to do so.
2. You may use rulers, compasses and paper for rough work.
3. Be sure that you understand the coding system for your response form. If you are not sure, ask your teacher
to clarify it. All coding must be done with a pencil, preferably HB. Fill in circles completely.
4. On your response form, print your school name, city/town, and province in the box in the upper right corner.
5. Be certain that you code your name, age, sex, grade, and the contest you are writing on the response
form. Only those who do so can be counted as official contestants.
6. This is a multiple-choice test. Each question is followed by five possible answers marked A, B, C, D, and
E. Only one of these is correct. When you have decided on your choice, fill in the appropriate circles on
the response form.
7. Scoring: Each correct answer is worth 5 credits in Part A, 6 credits in Part B, and 8 credits in Part C.
There is no penalty for an incorrect answer.
Each unanswered question is worth 2 credits, to a maximum of 20 credits.
8. Diagrams are not drawn to scale. They are intended as aids only.
9. When your supervisor instructs you to begin, you will have sixty minutes of working time.
Scoring: There is no penalty for an incorrect answer.
Each unanswered question is worth 2 credits, to a maximum of 20 credits.

Part A: Each question is worth 5 credits.

4×4+4
1. The value of is
2×2–2
(A) 2 (B) 6 (C) 10 (D) 12 (E) 18

2. ( )
If k = 2 , then k 3 – 8 (k + 1) equals

(A) 0 (B) 3 (C) 6 (D) 8 (E) – 6

3. If 4(♥)2 = 144 , then a value of ♥ is

(A) 3 (B) 6 (C) 9 (D) 12 (E) 18

4. (
Which of the following numbers divide exactly into 15 + 49 ? )
(A) 3 (B) 4 (C) 5 (D) 7 (E) 11

5. If 10% of 400 is decreased by 25, the result is


(A) 15 (B) 37.5 (C) 65 (D) 260 (E) 3975

6. In the diagram, a + b equals

(A) 10 (B) 85 (C) 110


(D) 170 (E) 190 110°
a° b°
60°

7. If 2 x – 1 = 5 and 3 y + 2 = 17 , then the value of 2 x + 3 y is

(A) 8 (B) 19 (C) 21 (D) 23 (E) 25

8. The average of four test marks was 60. The first three marks were 30, 55 and 65. What was the fourth
mark?

(A) 40 (B) 55 (C) 60 (D) 70 (E) 90

9. In the diagram, each small square is 1 cm by 1 cm. The area


of the shaded region, in square centimetres, is
(A) 2.75 (B) 3 (C) 3.25
(D) 4.5 (E) 6
10. 10 + 103 equals

(A) 2.0 × 103 (B) 8.0 × 103 (C) 4.0 × 101 (D) 1.0 × 10 4 (E) 1.01 × 103

Part B: Each question is worth 6 credits.

11. Today is Wednesday. What day of the week will it be 100 days from now?
(A) Monday (B) Tuesday (C) Thursday (D) Friday (E) Saturday

12. The time on a digital clock is 5:55. How many minutes will pass before the clock next shows a time
with all digits identical?
(A) 71 (B) 72 (C) 255 (D) 316 (E) 436

13. In Circle Land, the numbers 207 and 4520 are shown in the following way:

2 4
5
7 2

207 4520

In Circle Land, what number does the following diagram represent?

3
1
5

(A) 30 105 (B) 30 150 (C) 3105 (D) 3015 (E) 315

14. An 8 cm cube has a 4 cm square hole cut through its centre, 4 cm


cm

as shown. What is the remaining volume, in cm 3 ?


4

(A) 64 (B) 128 (C) 256


(D) 384 (E) 448 8 cm

8 cm
8 cm

15. For how many different values of k is the 4-digit number 7k 52 divisible by 12?
(A) 0 (B) 1 (C) 2 (D) 3 (E) 4

16. In an election, Harold received 60% of the votes and Jacquie received all the rest. If Harold won by
24 votes, how many people voted?

(A) 40 (B) 60 (C) 72 (D) 100 (E) 120


17. In the parallelogram, the value of x is x°
(A) 30 (B) 50 (C) 70
(D) 80 (E) 150
150°

80°

18. In the diagram, AD < BC . What is the perimeter of ABCD? A 7 D

(A) 23 (B) 26 (C) 27


(D) 28 (E) 30 4 5

B C

19. The numbers 49, 29, 9, 40, 22, 15, 53, 33, 13, 47 are grouped in pairs so that the sum of each pair is
the same. Which number is paired with 15?
(A) 33 (B) 40 (C) 47 (D) 49 (E) 53

( )( )(
20. The units (ones) digit in the product (5 + 1) 53 + 1 56 + 1 512 + 1 is )
(A) 6 (B) 5 (C) 2 (D) 1 (E) 0

Part C: Each question is worth 8 credits.

21. A number is Beprisque if it is the only natural number between a prime number and a perfect square
(e.g. 10 is Beprisque but 12 is not). The number of two-digit Beprisque numbers (including 10) is
(A) 1 (B) 2 (C) 3 (D) 4 (E) 5

22. If w = 2129 × 381 × 5128 , x = 2127 × 381 × 5128 , y = 2126 × 382 × 5128 , and z = 2125 × 382 × 5129 , then
the order from smallest to largest is
(A) w, x, y, z (B) x, w, y, z (C) x, y, z, w (D) z, y, x, w (E) x, w, z, y

23. Al and Bert must arrive at a town 22.5 km away. They have one bicycle between them and must arrive
at the same time. Bert sets out riding at 8 km/h, leaves the bicycle and then walks at 5 km/h. Al walks
at 4 km/h, reaches the bicycle and rides at 10 km/h. For how many minutes was the bicycle not in
motion?
(A) 60 (B) 75 (C) 84 (D) 94 (E) 109

24. A number is formed using the digits 1, 2, ..., 9. Any digit can be used more than once, but adjacent digits
cannot be the same. Once a pair of adjacent digits has occurred, that pair, in that order, cannot be used
again. How many digits are in the largest such number?

(A) 72 (B) 73 (C) 144 (D) 145 (E) 91

continued ...
25. Two circles C1 and C2 touch each other externally and the m
line l is a common tangent. The line m is parallel to l and C3
touches the two circles C1 and C3 . The three circles are C1
mutually tangent. If the radius of C2 is 9 and the radius of
C3 is 4, what is the radius of C1? C2
l
(A) 10.4 (B) 11 (C) 8 2
(D) 12 (E) 7 3
Canadian
Mathematics
Competition
An activity of The Centre for Education
in Mathematics and Computing,
University of Waterloo, Waterloo, Ontario

1999 Solutions
Pascal Contest(Grade 9)
for the

Awards

© 1999 Waterloo Mathematics Foundation


1999 Pascal Solutions 2

Part A

4×4+4
1. The value of is
2×2–2
(A) 2 (B) 6 (C) 10 (D) 12 (E) 18

Solution
4 × 4 + 4 16 + 4 20
= = = 10 ANSWER: (C)
2×2–2 4–2 2

2. ( )
If k = 2 , then k 3 – 8 (k + 1) equals

(A) 0 (B) 3 (C) 6 (D) 8 (E) – 6

Solution
( )
For k = 2 , k 3 – 8 (k + 1)

(
= 2 – 8 (2 + 1)
3
)
= 0(3)
=0 ANSWER: (A)

3. If 4(♥)2 = 144 , then a value of ♥ is

(A) 3 (B) 6 (C) 9 (D) 12 (E) 18

Solution
4(♥)2 = 144
♥2 = 36
♥ = ±6 ANSWER: (B)

4. (
Which of the following numbers divide exactly into 15 + 49 ? )
(A) 3 (B) 4 (C) 5 (D) 7 (E) 11

Solution
15 + 49 = 15 + 7 = 22
The only integer listed that divides 22 evenly is 11. ANSWER: (E)

5. If 10% of 400 is decreased by 25, the result is

(A) 15 (B) 37.5 (C) 65 (D) 260 (E) 3975


Solution
(10% of 400) – 25 = 40 – 25 = 15 . ANSWER: (A)
1999 Pascal Solutions 3

6. In the diagram, a + b equals

(A) 10 (B) 85 (C) 110


110°
(D) 170 (E) 190
a° b°
60°

Solution
The number of degrees at the centre of a circle is 360.
Thus, a + b + 110 + 60 = 360 (measured in degrees).
Therefore a + b = 190. ANSWER: (E)

7. If 2 x – 1 = 5 and 3 y + 2 = 17 , then the value of 2 x + 3 y is

(A) 8 (B) 19 (C) 21 (D) 23 (E) 25

Solution
2x – 1 = 5 , 3 y + 2 = 17
2x = 6 3 y = 15
Thus, 2 x + 3 y = 6 + 15 = 21. ANSWER: (C)
Note: It is not necessary to solve the equations to find actual values for x and y although this
would of course lead to the correct answer. It is, however, a little more efficient to solve for 2x and
3y.

8. The average of four test marks was 60. The first three marks were 30, 55 and 65. What was the
fourth mark?

(A) 40 (B) 55 (C) 60 (D) 70 (E) 90

Solution
The total number of marks scored on the four tests was 4 × 60 or 240. The total number of marks
scored on the first three tests was 150. The fourth mark was 240 – 150 = 90 . ANSWER: (E)

9. In the diagram, each small square is 1 cm by 1 cm. The


area of the shaded region, in square centimetres, is

(A) 2.75 (B) 3 (C) 3.25


(D) 4.5 (E) 6

Solution
The shaded triangle has a base of 2 cm and a height of 3 cm.
1999 Pascal Solutions 4

2×3
Its area is = 3 (sq. cm). ANSWER: (B)
2

10. 10 + 103 equals

(A) 2.0 × 103 (B) 8.0 × 103 (C) 4.0 × 101 (D) 1.0 × 10 4 (E) 1.01 × 103

Solution
10 + 103 = 10 + 1000 = 1010 = 1.01 × 103 ANSWER: (E)

Part B

11. Today is Wednesday. What day of the week will it be 100 days from now?

(A) Monday (B) Tuesday (C) Thursday (D) Friday (E) Saturday

Solution
Since there are 7 days in a week it will be Wednesday in 98 days.
In 100 days it will thus be Friday. ANSWER: (D)

12. The time on a digital clock is 5:55. How many minutes will pass before the clock next shows a time
with all digits identical?

(A) 71 (B) 72 (C) 255 (D) 316 (E) 436

Solution
The digits on the clock will next be identical at 11:11. This represents a time difference of 316
minutes. (Notice that times like 6:66, 7:77 etc. are not possible.) ANSWER: (D)

13. In Circle Land, the numbers 207 and 4520 are shown in the following way:

4
2 5
7 2
207 4520

In Circle Land, what number does the following diagram represent?

3
1
5
1999 Pascal Solutions 5

(A) 30 105 (B) 30 150 (C) 3105 (D) 3015 (E) 315

Solution 1
= 3 × 10 4 = 30 000
3

1 = 1 × 10 2 = 100

5 = 5 × 10 0 = 5
The required number is 30 000 + 100 + 5 = 30 105.

Solution 2
Since there are four circles around the ‘3’ this corresponds to 3 × 10 4 = 30 000 .
The ‘5’ corresponds to a 5 in the units digit which leads to 30 105 as the only correct possibility.
ANSWER: (A)

4 cm
14. An 8 cm cube has a 4 cm square hole cut through its centre,

cm
as shown. What is the remaining volume, in cm 3 ?

4
(A) 64 (B) 128 (C) 256
(D) 384 (E) 448 8 cm

8 cm
8 cm

Solution
Remaining volume = 8 × 8 × 8 – 8 × 4 × 4 (in cm 3 )
= 8(64 – 16)
= 8 × 48
= 384 ANSWER: (D)

15. For how many different values of k is the 4-digit number 7k 52 divisible by 12?

(A) 0 (B) 1 (C) 2 (D) 3 (E) 4

Solution
Since 12 = 4 × 3 the number 7k 52 must be divisible by both 4 and 3. Since 52 is the number formed
by the last two digits divisible by 4 then we need only ask, ‘for what values of k is 7k 52 divisible by
3?’ If a number is divisible by 3 the sum of its digits must be a multiple of 3. Thus 7 + k + 5 + 2 or
14 + k must be a multiple of 3. The only acceptable values for k are 1, 4 or 7.
Thus, are three values. ANSWER: (D)
1999 Pascal Solutions 6

16. In an election, Harold received 60% of the votes and Jacquie received all the rest. If Harold won by
24 votes, how many people voted?

(A) 40 (B) 60 (C) 72 (D) 100 (E) 120

Solution
If Harold received 60% of the votes this implies that Jacquie received 40% of the total number of
votes. The difference between them, 20%, represents 24 votes.
Therefore, the total number of votes cast was 5 × 24 = 120 . ANSWER: (E)

17. In the parallelogram, the value of x is x°


(A) 30 (B) 50 (C) 70
(D) 80 (E) 150
150°

80°

Solution
The angle in the parallelogram opposite the angle measuring 80° is also 80°. The angle
supplementary to 150° is 30°.
In the given triangle we now have, x° + 80° + 30° = 180° .
Therefore x = 70. ANSWER: (C)

18. In the diagram, AD < BC . What is the perimeter of


ABCD? A 7 D

(A) 23 (B) 26 (C) 27 5


4
(D) 28 (E) 30

B C

Solution
From D we draw a line perpendicular to BC that meets A 7 D
BC at N. Since ADNB is a rectangle and AD BC ,
DN = 4. We use Pythagoras to find NC = 3. We now 5
4 4
know that BC = BN + NC = 7 + 3 = 10.
The required perimeter is 7 + 5 + 10 + 4 = 26 .
B N C
ANSWER: (B)
1999 Pascal Solutions 7

19. The numbers 49, 29, 9, 40, 22, 15, 53, 33, 13, 47 are grouped in pairs so that the sum of each pair is
the same. Which number is paired with 15?

(A) 33 (B) 40 (C) 47 (D) 49 (E) 53

Solution
If we arrange the numbers in ascending order we would have: 9, 13, 15, 22, 29, 33, 40, 47, 49, 53. If
the sum of each pair is equal they would be paired as: 9 ↔ 53 , 13 ↔ 49 , 15 ↔ 47 , 22 ↔ 40 ,
29 ↔ 33 . ANSWER: (C)

20. ( )( )(
The units (ones) digit in the product (5 + 1) 53 + 1 56 + 1 512 + 1 is )
(A) 6 (B) 5 (C) 2 (D) 1 (E) 0

Solution
We start by observing that each of 53 , 56 and 512 have a units digit of 5. This implies that each of
5 + 1 , 53 + 1, 56 + 1 and 512 + 1 will then have a units digit of 6.
If we multiply any two numbers having a units digit of 6, their product will also have a units digit of
6. Applying this to the product of four numbers, we see that the final units digit must be a 6.
ANSWER: (A)

Part C

21. A number is Beprisque if it is the only natural number between a prime number and a perfect square
(e.g. 10 is Beprisque but 12 is not). The number of two-digit Beprisque numbers (including 10) is

(A) 1 (B) 2 (C) 3 (D) 4 (E) 5

Solution
We start with the observation that it is necessary to consider only the odd perfect squares and the
integers adjacent to them. It is not necessary to consider the even perfect squares because if we add 2
or subtract 2 from an even number the result is even and it is required by the conditions set out in the
question that this number be prime. Considering then the odd perfect squares we have: {9, 10 , 11} ,
{23, 24 , 25, 26, 27} , {47, 48 , 49, 50, 51} , {79, 80 , 81, 82 , 83} .
The Beprisque numbers are those that are circled. ANSWER: (E)

22. If w = 2129 × 381 × 5128 , x = 2127 × 381 × 5128 , y = 2126 × 382 × 5128 , and z = 2125 × 382 × 5129 , then
the order from smallest to largest is

(A) w, x, y, z (B) x, w, y, z (C) x, y, z, w (D) z, y, x, w (E) x, w, z, y

Solution
We start with the observation that 2125 × 381 × 5128 is a common factor to each of the given numbers.
1999 Pascal Solutions 8

For the basis of comparison, we remove the common factor and write the numbers as follows:
( )
w = 2 4 ⋅ 53 2125 × 381 × 5128 = 2000 k

x = 2 2 ⋅ 53 (2125 × 381 × 5128 ) = 500 k

y = 2 ⋅ 3 ⋅ 53 (2125 × 381 × 5128 ) = 750 k


z = 3 ⋅ 54 (2125 × 381 × 5128 ) = 1875k , where k = 2125 × 381 × 5128 .
Thus, x < y < z < w . ANSWER: (C)

23. Al and Bert must arrive at a town 22.5 km away. They have one bicycle between them and must
arrive at the same time. Bert sets out riding at 8 km/h, leaves the bicycle and then walks at 5 km/h.
Al walks at 4 km/h, reaches the bicycle and rides at 10 km/h. For how many minutes was the
bicycle not in motion?

(A) 60 (B) 75 (C) 84 (D) 94 (E) 109

Solution
Let x represent the distance that Bert rides his bicycle.
Therefore, he walks for (22.5 – x ) km.
Bert’s total time for the trip is  +
x 22.5 – x 
hours and Al’s is  +
x 22.5 – x 
hours.
8 5   4 10 
Since their times are equal,
x 22.5 – x x 22.5 – x
+ = +
8 5 4 10
22.5 – x 22.5 – x x x
– = –
5 10 4 8
2(22.5 – x ) 22.5 – x 2 x x
– = –
10 10 8 8
22.5 – x x
=
10 8
10 x = 180 – 8 x
18 x = 180
x = 10.
This means that Bert rode for 1.25 h before he left the bicycle and Al walked for 2.5 h before he
picked it up. The bicycle was thus not in motion for 1.25 h or 75 minutes. ANSWER: (B)

24. A number is formed using the digits 1, 2, ..., 9. Any digit can be used more than once, but adjacent
digits cannot be the same. Once a pair of adjacent digits has occurred, that pair, in that order, cannot
be used again. How many digits are in the largest such number?

(A) 72 (B) 73 (C) 144 (D) 145 (E) 91


1999 Pascal Solutions 9

Solution
Since there are 9(8) = 72 ordered pairs of consecutive digits, and since the final digit has no
successor, we can construct a 73 digit number by adding a 9. The question is, of course, can we
actually construct this number? The answer is ‘yes’ and the largest such number is,
98 97 96 95 94 93 92 91 87 86 85 84 83 82 81 76 75 74 73 72 71 65 64 63 62 61
54 53 52 51 43 42 41 32 31 21 9.
If we count the numbers in the string we can see that there are actually 73 numbers contained within
it. ANSWER: (B)

25. Two circles C1 and C2 touch each other externally and the m
C3
line l is a common tangent. The line m is parallel to l and
touches the two circles C1 and C3 . The three circles are C1
mutually tangent. If the radius of C2 is 9 and the radius of
C3 is 4, what is the radius of C1? C2
l
(A) 10.4 (B) 11 (C) 8 2
(D) 12 (E) 7 3

Solution
We start by joining the centres of the circles to form m
∆ C1C2 C3 . (The lines joining the centres pass through the A C3
B
corresponding points of tangency.) C1
Secondly, we construct the rectangle ABC2 D as shown in
the diagram. If the radius of the circle with centre C1 is r D C2
we see that: C1C2 = r + 9 , C1C3 = r + 4 and C2 C3 = 13. l

We now label lengths on the rectangle in the way noted in A C3 B


the diagram. r –4 r+4
C1 13 2r – 13
r+9
r –9
D C2

To understand this labelling, look for example at C1D . The radius of the large circle is r and the
radius of the circle with centre C2 is 9. The length C1D is then r – 9 .
This same kind of reasoning can be applied to both C1 A and BC2 .

Using Pythagoras we can now derive the following:


In ∆ AC3C1 , C3 A2 = (r + 4)2 – (r – 4)2
= 16r .
Therefore C3 A = 4 r .
1999 Pascal Solutions 10

In ∆ DC1C2 , ( DC2 )2 = (r + 9)2 – (r – 9)2


= 36r.
Therefore DC2 = 6 r .

In ∆ BC3C2 , (C3 B)2 = 132 – (2r – 13)2


= – 4r 2 + 52r .
Therefore C3 B = – 4r 2 + 52r .
In a rectangle opposite sides are equal, so:
DC2 = C3 A + C3 B
or, 6 r = 4 r + – 4r 2 + 52r
2 r = – 4r 2 + 52r .

Squaring gives, 4r = – 4r 2 + 52r


4r 2 – 48r = 0
4r(r – 12) = 0
Therefore r = 0 or r = 12.
Since r > 0 , r = 12. ANSWER: (D)

Anda mungkin juga menyukai